Collaboration of Practice Questions - NEURO EXAM #3

Lakukan tugas rumah & ujian kamu dengan baik sekarang menggunakan Quizwiz!

An 18-year-old client was hit in the head with a baseball during practice. When discharging him to the care of his mother, the nurse gives which of the following instructions?

"Wake him every hour and assess his orientation to person, time, and place." Changes in LOC may indicate expanding lesions such as subdural hematoma; orientation and LOC are assessed frequently for 24 hours. A keyhole pupil is found after iridectomy. Profuse or projectile vomiting is a symptom of increased ICP and should be reported immediately. A slight headache may last for several days after concussion; severe or worsening headaches should be reported.

What health history question will give the nurse the most information when evaluating a patient for Guillain-Barré syndrome (GBS)? 1. "Have you had a respiratory virus in the past 2 weeks?" 2. "Have you ever been exposed to Epstein-Barr virus?" 3. "Has anyone else in your family ever had GBS?" 4. "Did you get a flu vaccine in the past year?"

1

A client diagnosed with a subarachnoid hemorrhage has undergone a craniotomy for repair of a ruptured aneurysm. Which intervention will the intensive care nurse implement? 1. Administer a stool softener b.i.d. 2. Encourage the client to cough hourly. 3. Monitor neurological status every shift. 4. Maintain the dopamine drip to keep BP at 160/90.

1 The client is at risk for increased intracranial pressure whenever performing the Valsalva maneuver, which will occur when straining during defecation. Therefore, stool softeners would be appropriate.

The nurse is assessing the client diagnosed with meningococcal meningitis. Which assessment data would warrant notifying the HCP?1. Purpuric lesions on the face. 2. Complaints of light hurting the eyes. 3. Dull, aching, frontal headache. 4. Not remembering the day of the week.

1. In clients with meningococcal meningitis, purpuric lesions over the face and extremity are the signs of a fulminating infection that can lead to death within a few hours. 2. common 3. would be expected 4. wouldn't warrant PCP

hich assessment intervention should the nurse implement specifically for the diagnosis of Guillain-Barré syndrome? 1. Assess deep tendon reflexes. 2. Complete a Glasgow Coma Scale. 3. Check for Babinski's reflex. 4. Take the client's vital signs.

1. Hyporeflexia of the lower extremities is the classic clinical manifestation of this syndrome. Therefore, assessing deep tendon reflexes is appropriate.

The client diagnosed with septic meningitis is admitted to the medical floor at noon. Which health-care provider's order would have the highest priority? 1. Administer an intravenous antibiotic. 2. Obtain the client's lunch tray. 3. Provide a quiet, calm, and dark room.

1. The antibiotic has the highest priority because failure to treat a bacterial infection can result in shock, systemic sepsis, and death.

The client has undergone a craniotomy for a brain tumor. Which data indicate a complication of this surgery? 1. The client complains of a headache at "3" to "4" on a 1-to-10 scale. 2. The client has an intake of 1,000 mL and an output of 3,500 mL. 3. The client complains of a raspy sore throat. 4. The client experiences dizziness when trying to get up too quickly.

2 An output much larger than the intake could indicate the development of diabetes insipidus. Pressure on the pituitary gland can result in decreased production of vasopressin, the antidiuretic hormone.

The rehabilitation nurse caring for the client with an L1 SCI is developing the nursing care plan. Which intervention should the nurse implement? 1. Keep oxygen via nasal cannula on at all times. 2. Administer low-dose subcutaneous anticoagulants. 3. Perform active lower extremity ROM exercises. 4. Refer to a speech therapist for ventilator-assisted speech

2 Deep vein thrombosis (DVT) is a potential complication of immobility, which can occur because the client cannot move the lower extremities as a result of the L1 SCI. Low-dose anticoagulation therapy (Lovenox) helps prevent blood from coagulating, thereby preventing DVTs.

The client is being admitted to rule out a brain tumor. Which classic triad of symptoms supports a diagnosis of brain tumor? 1. Nervousness, metastasis to the lungs, and seizures. 2. Headache, vomiting, and papilledema. 3. Hypotension, tachycardia, and tachypnea. 4. Abrupt loss of motor function, diarrhea, and changes in taste.

2 The classic triad of symptoms suggesting a brain tumor includes a headache that is dull, unrelenting, and worse in the morning; vomiting unrelated to food intake; and edema of the optic nerve (papilledema), which occurs in 70% to 75% of clients diagnosed with brain tumors. Papilledema causes visual disturbances such as decreased visual acuity and diplopia.

The nurse arrives at the site of a one-car motor-vehicle accident and stops to render aid. The driver of the car is unconscious. After stabilizing the client's cervical spine, which action should the nurse take next? 1. Carefully remove the driver from the car. 2. Assess the client's pupils for reaction. 3. Assess the client's airway. 4. Attempt to wake the client up by shaking him.

3 The nurse must maintain a patent air- way. Airway is the first step in resusci- tation.

The nurse and an unlicensed assistive personnel (UAP) are caring for a client with right-sided paralysis. Which action by the UAP requires the nurse to intervene? 1. The assistant places a gait belt around the client's waist prior to ambulating. 2. The assistant places the client on the back with the client's head to the side. 3. The assistant places a hand under the client's right axilla to move up in bed. 4. The assistant praises the client for attempting to perform ADLs independently.

3 This action is inappropriate and would require intervention by the nurse because pulling on a flaccid shoulder joint could cause shoulder dislocation; the client should be pulled up by placing the arm underneath the back or using a lift sheet.

Which assessment data would indicate to the nurse that the client would be at risk for a hemorrhagic stroke? 1. A blood glucose level of 480 mg/dL. 2. A right-sided carotid bruit. 3. A blood pressure of 220/120 mm Hg. 4. The presence of bronchogenic carcinoma.

3 Uncontrolled hypertension is a risk factor for hemorrhagic stroke, whichis a ruptured blood vessel inside the cranium.

The client diagnosed with Guillain-Barré syndrome is having difficulty breathing and is placed on a ventilator. Which situation warrants immediate intervention by the nurse? 1. The ventilator rate is set at 14 breaths per minute. 2. A manual resuscitation bag is at the client's bedside .3. The client's pulse oximeter reading is 85%. 4. The ABG results are pH 7.40, PaO2 88, PaCO2 35, and HCO3 24.

3. A pulse oximeter reading of less than 93% warrants immediate intervention; a 90% peripheral oxygen saturation indicates a PaO2 of about 60 (normal, 80 to 100). When the client is placed on the ventilator, this should cause the client's oxygen level to improve.

The wife of the client diagnosed with septic meningitis asks the nurse, "I am so scared. What is meningitis?" Which statement would be the most appropriate response by the nurse? 1. "There is bleeding into his brain causing irritation of the meninges." 2. "A virus has infected the brain and meninges, causing inflammation." 3. "This is a bacterial infection of the tissues that cover the brain and spinal cord." 4. "This is an inflammation of the brain parenchyma caused by a mosquito bite."

3. Septic meningitis refers to meningitis caused by bacteria; the most common form of bacterial meningitis is caused by the Neisseria meningitides bacteria. (1 &2 - aseptic - virus)

The client has been diagnosed with a cerebrovascular accident (stroke). The client's wife is concerned about her husband's generalized weakness. Which home modification should the nurse suggest to the wife prior to discharge? 1. Obtain a rubber mat to place under the dinner plate. 2. Purchase a long-handled bath sponge for showering. 3. Purchase clothes with Velcro closure devices. 4. Obtain a raised toilet seat for the client's bathroom.

4 Raising the toilet seat is modifying the home and addresses the client's weakness in being able to sit down and get up without straining muscles or requiring lifting assistance from the wife.

The client admitted with rule-out Guillain-Barré syndrome has just had a lumbar puncture. Which intervention should the nurse implement postprocedure? 1. Monitor the client for hypotension. 2. Apply pressure to the puncture site. 3. Test the client's cerebrospinal fluid. 4. Increase the client's fluid intake.

4. increased fluid intake will help prevent a postprocedure headache, which may occur after a lumbar puncture.

The client diagnosed with a brain tumor was admitted to the intensive care unit with decorticate posturing. Which indicates that the client's condition is becoming worse? 1. The client has purposeful movement with painful stimuli. 2. The client has assumed adduction of the upper extremities. 3. The client is aimlessly thrashing in the bed. 4. The client has become flaccid and does not respond to stimuli.

4 The most severe neurological impairment result is flaccidity and no response to stimuli. This indicates that the client's condition has worsened.

Which of the following clients on the rehab unit is most likely to develop autonomic dysreflexia?

A client with a high cervical spine injury Autonomic dysreflexia refers to uninhibited sympathetic outflow in clients with spinal cord injuries about the level of T10. The other clients aren't prone to dysreflexia.

A client is admitted to the critical care unit with possible GBS. Which symptom of neurologic impairment will require priority nursing interventions? Select all that apply. A. New adventitious breath sounds B. A RR of 12 C. Rapid, shallow breathing pattern D. A peripheral oxygen saturation of 90% E. New-onset nausea following a position change

A, C, D

A client has a subarachnoid bolt. What action by the nurse is most important? a. Balancing and recalibrating the device b. Documenting intracranial pressure readings c. Handling the fiberoptic cable with care to avoid breakage d. Monitoring the clients phlebostatic axis

ANS: A This device needs frequent balancing and recalibration in order to read correctly. Documenting readings is important, but it is more important to ensure the devices accuracy. The fiberoptic transducer-tipped catheter has a cable that must be handled carefully to avoid breaking it, but ensuring the devices accuracy is most important. The phlebostatic axis is not related to neurologic monitoring.

The nurse reviews laboratory data for a client who has Guillain-Barré syndrome (GBS). Which result does the nurse correlate with this disease process? a.Increased cerebrospinal fluid (CSF) protein level b.Decreased serum protein electrophoresis results c.Increased antinuclear antibodies d.Decreased immune globulin G (IgG) levels

ANS: A A lumbar puncture is performed to evaluate the CSF. An increased CSF protein level without increased cell count is a distinguishing feature of GBS. The other results are not associated with GBS.

The nurse teaches a client who has autonomic dysfunction about injury prevention. Which statement indicates that the client correctly understands the teaching? a."I will change positions slowly." b."I will avoid wearing cotton socks." c."I will use an electric razor." d."I will use a heating pad on my feet."

ANS: A Autonomic dysfunction causes orthostatic hypotension. The client should change positions slowly to prevent orthostatic hypotension. Autonomic dysfunction can cause peripheral polyneuropathy, so the client should be taught to wear socks and shoes at all times and not to use a heating pad. The disorder does not cause bleeding; therefore the client can use any type of razor.

An older client is hospitalized with Guillain-Barré syndrome. A family member tells the nurse the client is restless and seems confused. What action by the nurse is best? a. Assess the client's oxygen saturation. b. Check the medication list for interactions. c. Place the client on a bed alarm. d. Put the client on safety precautions.

ANS: A In the older adult, an early sign of hypoxia is often confusion and restlessness. The nurse should first assess the client's oxygen saturation. The other actions are appropriate, but only after this assessment occurs.

A client had an embolic stroke and is having an echocardiogram. When the client asks why the provider ordered a test on my heart, how should the nurse respond? a. Most of these types of blood clots come from the heart. b. Some of the blood clots may have gone to your heart too. c. We need to see if your heart is strong enough for therapy. d. Your heart may have been damaged in the stroke too.

ANS: A An embolic stroke is caused when blood clots travel from one area of the body to the brain. The most common source of the clots is the heart. The other statements are inaccurate.

The nurse caring for a client with Guillain-Barr syndrome has identified the priority client problem of decreased mobility for the client. What actions by the nurse are best? (Select all that apply.) a. Ask occupational therapy to help the client with activities of daily living. b. Consult with the provider about a physical therapy consult. c. Provide the client with information on support groups. d. Refer the client to a medical social worker or chaplain. e. Work with speech therapy to design a high-protein diet.

ANS: A, B, E Improving mobility and strength involves the collaborative assistance of occupational therapy, physical therapy, and speech therapy. While support groups, social work, or chaplain referrals may be needed, they do not help with mobility.

The nurse caring for a client with Guillain-Barré syndrome has identified the priority client problem of decreased mobility for the client. What actions by the nurse are best? (Select all that apply.) a. Ask occupational therapy to help the client with activities of daily living. b. Consult with the provider about a physical therapy consult. c. Provide the client with information on support groups. d. Refer the client to a medical social worker or chaplain. e. Work with speech therapy to design a high-protein diet.

ANS: A, B, E Improving mobility and strength involves the collaborative assistance of occupational therapy, physical therapy, and speech therapy. While support groups, social work, or chaplain referrals may be needed, they do not help with mobility.

A nurse cares for older clients who have traumatic brain injury. What should the nurse understand about this population? (Select all that apply.) a. Admission can overwhelm the coping mechanisms for older clients. b. Alcohol is typically involved in most traumatic brain injuries for this age group. c. These clients are more susceptible to systemic and wound infections. d. Other medical conditions can complicate treatment for these clients. e. Very few traumatic brain injuries occur in this age group.

ANS: A, C, D Older clients often tolerate stress poorly, which includes being admitted to a hospital that is unfamiliar and noisy. Because of decreased protective mechanisms, they are more susceptible to both local and systemic infections. Other medical conditions can complicate their treatment and recovery. Alcohol is typically not related to traumatic brain injury in this population; such injury is most often from falls and motor vehicle crashes. The 65- to 76-year-old age group has the second highest rate of brain injuries compared to other age groups.

The nurse is caring for a client diagnosed with Guillain Barre syndrome. Which assessment findings require nursing action? (Select all that apply.) a. Blood pressure of 80/42 b. A respiratory rate of 24 c. Shallow breathing pattern d. A peripheral oxygen saturation (SpO2) of 85% e. Diminished breath sounds in all lung fields

ANS: A, C, D, E All choices except B are abnormal assessment findings that can occur in clients with this disease. A respiratory rate of 24 is slightly elevated but does not require nursing action.

The nurse recognizes which pathophysiologic feature as a hallmark of Guillain-Barré syndrome? a.Nerve impulses are not transmitted to skeletal muscle. b.The immune system destroys the myelin sheath. c.The distal nerves degenerate and retract. d.Antibodies to acetylcholine receptor sites develop.

ANS: B In Guillain-Barré syndrome, the immune system destroys the myelin sheath, causing segmental demyelination. Nerve impulses are transmitted more slowly but remain in place. Antibodies are not developed. The nerves do not degenerate and retract.

The intensive care nurse is caring for a client who has Guillain-Barré syndrome. The nurse notes that the client's vital capacity has declined to 12 mL/kg, and the client is having difficulty clearing secretions. Which is the nurse's priority action? a.Place the client in a high Fowler's position. b.Prepare the client for elective intubation. c.Administer oxygen via a nasal cannula. d.Auscultate for breath sounds.

ANS: B Deterioration in vital capacity to less than 15 mL/kg and an inability to clear secretions are indications for elective intubation. The other interventions may assist with breathing and oxygenation but would not reverse the deterioration in vital capacity or help clear secretions.

A client returns from the postanesthesia care unit (PACU) after a craniotomy for removal of a left parietal lobe tumor. How will the nurse position the client after surgery? A. Flex the client's knees to decrease intra-abdominal pressure and cerebral hypertension. B. Keep the client on the left side to prevent surgical site bleeding or cerebrospinal fluid leakage. C. Elevate the client's head to at least 30 degrees to promote cerebral venous drainage. D. Hyperextend the client's neck to maintain the airway and prevent aspiration regardless of supine or side-lying positioning.

ANS: C Rationale: Elevation of the backrest allows both CSF and cerebral venous blood to drain out of the cranium. Avoid placing a client who has undergone a craniotomy on the operative side. Avoid hip and knee flexion because this increases intracranial pressure, and increased intracranial pressure from edema is common in clients after cranial surgery. Hyperextension of the neck will reduce CSF and venous outflow from the cranium.

A client with Guillain-Barré syndrome is admitted to the hospital. The nurse plans caregiving priority to interventions that address which priority client problem? a. Anxiety b. Low fluid volume c. Inadequate airway d. Potential for skin breakdown

ANS: C Airway takes priority. Anxiety is probably present, but a physical diagnosis takes priority over a psychosocial one. The client has no reason to have low fluid volume unless he or she has been unable to drink for some time. If present, airway problems take priority over a circulation problem. An actual problem takes precedence over a risk for a problem.

The nurse teaches a client who has Guillain-Barré syndrome (GBS) about pain management. Which statement indicates that the client correctly understands the teaching? a."I can use the button on the pump as often as I want to get more pain medication." b."Aspirin will provide the best relief from my pain associated with this disease." c."A combination of morphine and distraction helps bring me relief right now." d."I should not have any pain as a result of impaired motor and sensory neurons."

ANS: C Typical pain from GBS often is not relieved by medication other than opiates. Distraction, repositioning, massage, heat, cold, and guided imagery may enhance the opiate effects. Patient-controlled analgesia (PCA) pumps should be set with appropriate doses and limits.

The nurse is evaluating the collaborative care of a client with traumatic brain injury (TBI). What is the most important goal for this client? a. Increasing cerebral perfusion b. Preventing further injury c. Preventing respiratory distress d. Achieving the highest level of functioning

ANS: D The nurse's goal for the client with TBI is to help him or her achieve the highest level of functioning possible. The nurse assesses cerebral perfusion, such as oxygenation status, but cannot increase cerebral perfusion. Prevention of injury from falls, infection, or further impairment of cerebral perfusion is part of a larger goal for this client. Prevention of respiratory distress is also part of a larger goal for this client.

A client is admitted with Guillain-Barr syndrome (GBS). What assessment takes priority? a. Bladder control b. Cognitive perception c. Respiratory system d. Sensory functions

ANS: CClients with GBS have muscle weakness, possibly to the point of paralysis. If respiratory muscles are paralyzed, the client may need mechanical ventilation, so the respiratory system is the priority. The nurse will complete urinary, cognitive, and sensory assessments as part of a thorough evaluation.

A client is admitted with a stroke (brain attack). Which tool does the nurse use to facilitate a focused neurologic assessment of the client? a. Mini-Mental State Examination (MMSE; mini-mental status examination) b. Intracranial pressure monitor c. Reflex hammer d. National Institutes of Health Stroke Scale (NIHSS)

ANS: D

A client recovering from a stroke reports double vision that is preventing the client from effectively completing activities of daily living. How does the nurse help the client compensate? a. Places objects in the client's field of vision b. Encourages turning the head from side to side c. Approaches the client on the affected side d. Covers the affected eye

ANS: D Covering the client's eye with a patch prevents diplopia. The client who is recovering from a stroke should always be approached on the unaffected side. The nurse may encourage side-to-side head turning for clients with hemianopsia (blindness in half of the visual field). Objects should be placed in the field of vision for the client with a decreased visual field.

A client with a confirmed acute ischemic stroke is comatose but breathing spontaneously. The client has an advance directive requesting limited resuscitation and is not a candidate for fibrinolytic therapy. What is the nurse's priority action on admission? A. Ask for palliative care consultation to assist with end-of-life decision making. B. Consult with the speech-language pathologist about alternative strategies for communication. C. Evaluate swallowing ability with an institution-specific, evidence-based protocol. D. Assess vital signs and determine if the advance directives need to be communicated to the health care provider.

ANS: D Rationale: Client values and preferences must be incorporated into high-quality care. Assessing VS will provide essential information about the urgency of obtaining an order for "Do not resuscitate" or variation provided by institutional policy. Communication with the health care provider is essential to ensure all health care team members share the client's preference about limited resuscitation and that those wishes are reflected in the medical record. Comatose clients are unresponsive; an alternate form of communication is not feasible. Palliative care can assist with managing symptoms and may be needed in subsequent days, but the end-of-life decisions have been made and are in the advanced directive.

A nurse receives a report on a client who had a left-sided stroke and has homonymous hemianopsia. What action by the nurse is most appropriate for this client? a. Assess for bladder retention and/or incontinence. b. Listen to the clients lungs after eating or drinking. c. Prop the clients right side up when sitting in a chair. d. Rotate the clients meal tray when the client stops eating.

ANS: D This condition is blindness on the same side of both eyes. The client must turn his or her head to see the entire visual field. The client may not see all the food on the tray, so the nurse rotates it so uneaten food is now within the visual field. This condition is not related to bladder function, difficulty swallowing, or lack of trunkcontrol.

The nurse learns that the pathophysiology of Guillain-Barr syndrome includes segmental demyelination. The nurse should understand that this causes what?a. Delayed afferent nerve impulses b. Paralysis of affected muscles c. Paresthesia in upper extremities d. Slowed nerve impulse transmission

ANS: D Demyelination leads to slowed nerve impulse transmission. The other options are not correct.

The nurse learns that the pathophysiology of Guillain-Barré syndrome includes segmental demyelination. The nurse should understand that this causes what? a. Delayed afferent nerve impulses b. Paralysis of affected muscles c. Paresthesia in upper extremities d. Slowed nerve impulse transmission

ANS: D Demyelination leads to slowed nerve impulse transmission. The other options are not correct.

A client with a stroke is being evaluated for fibrinolytic therapy. What information from the client or family is most important for the nurse to obtain? a. Loss of bladder control b. Other medical conditions c. Progression of symptoms d. Time of symptom onset

ANS: D The time limit for initiating fibrinolytic therapy for a stroke is 3 to 4.5 hours, so the exact time of symptom onset is the most important information for this client. The other information is not as critical.

The nurse is discussing the purpose of an electroencephalogram (EEG) with the family of a client with massive cerebral hemorrhage and loss of consciousness. It would be most accurate for the nurse to tell family members that the test measures which of the following conditions?

Activity of the brain An EEG measures the electrical activity of the brain. Extent of intracranial bleeding and location of the injury site would be determined by CT or MRI. Percent of functional brain tissue would be determined by a series of tests.

When discharging a client from the ER after a head trauma, the nurse teaches the guardian to observe for a lucid interval. Which of the following statements best described a lucid interval?

An interval when the client is oriented but then becomes somnolent. A lucid interval is described as a brief period of unconsciousness followed by alertness; after several hours, the client again loses consciousness. Garbled speech is known as dysarthria. An interval in which the client is alert but can't recall recent events is known as amnesia. Warning symptoms or auras typically occur before seizures.

When evaluating an ABG from a client with a subdural hematoma, the nurse notes the PaCO2 is 30 mm Hg. Which of the following responses best describes this result?

Appropriate; lowering carbon dioxide (CO2) reduces intracranial pressure (ICP). A normal PaCO2 value is 35 to 45 mm Hg. CO2 has vasodilating properties; therefore, lowering PaCO2 through hyperventilation will lower ICP caused by dilated cerebral vessels. Oxygenation is evaluated through PaO2 and oxygen saturation. Alveolar hypoventilation would be reflected in an increased PaCO2.

The nurse is caring for clients on a rehabilitation unit. Which nursing task would be most appropriate for the nurse to delegate to the unlicensed assistive personnel (UAP)?

Ask the UAP to hold the urinal while the client performs the Credé maneuver.

client returns from PACU after removal of a brainstem tumor. What position will the nurse place the client? a. Turn the patient from side to side to prevent aspiration. b. Keep the client flat in bed or up 10 degrees and reposition from side to side. c. Elevate the head of the bed to at least 30 degrees at all times. d. Keep the client in a sitting position in bed at all times.

B

A family member asks the nurse about whether there would be any long-term psychological effects from a client's mild traumatic brain injury. What is the nurse's best response? a. "You need to talk with the client's primary health care provider." b. "Usually any effects last for only a few weeks or months." c. "Each person's reaction to brain injury is different." d. "You should expect a change in the client's personality."

C

A client begins to have severe epistaxis after completing a dose of alteplase. In order of priority, what are the nurse's actions? A. Obtain vital signs. B. Assess the airway, and set up suction at bedside. C. Draw blood for anticoagulation studies. D. Call the health care provider.

B, A, D, C Epistasis is a severe nosebleed, and blood can block the oropharynx. The first priority is ensuring that airway and breathing are maintained. Next determine whether adequate circulation (VS) is present. Obtaining assistance from the rapid response team or health care provider to provide timely interventions (these may include oxygen, nasal packing, or imaging tests) to prevent complications follows immediate assessment of the airway, breathing, and circulation (ABCs). Drawing blood for anticoagulation studies will guide therapy, but it is not the immediate action required to avoid a life-threatening compromise to the respiratory system.

Which statements about stroke prevention indicate a client's understanding of health teaching by the nurse? (sata.) a. "I will take aspirin every day." b. "I have decided to stop smoking." c. "I will try to walk at least 30 minutes most days of the week." d. "I need to cut down a lot on my drinking." e. "I'm going to decrease salt in my diet."

B, C, D, E

A client with a spinal cord injury suddenly experiences an episode of autonomic dysreflexia. After checking the client's vital signs, list in order of priority, the nurse's actions (Number 1 being the first priority and number 5 being the last priority). A. Check for bladder distention B. Raise the head of the bed C. Contact the physician D. Loosen tight clothing on the client E. Administer an antihypertensive medication B, D, A, C, E.

B, D, A, C, E. Autonomic dysreflexia is characterized by severe hypertension, bradycardia, severe headache, nasal stuffiness, and flushing. The cause is a noxious stimulus, most often a distended bladder or constipation. Autonomic dysreflexia is a neurological emergency and must be treated promptly to prevent a hypertensive stroke. Immediate nursing actions are to sit the client up in bed in a high-Fowler's position and remove the noxious stimulus. The nurse should loosen any tight clothing and then check for bladder distention. If the client has a foley catheter, the nurse should check for kinks in the tubing. The nurse also would check for a fecal impaction and disimpact if necessary. The physician is contacted especially if these actions do not relieve the signs and symptoms. Antihypertensive medications may be prescribed by the physician to minimize cerebral hypertension.

The nurse is obtaining a health history for a 45-year-old woman with Guillain-Barré syndrome (GBS). Which statement by the client does the nurse correlate with the client's diagnosis? a."My neighbor also had Guillain-Barré syndrome." b."I had a viral infection about 2 weeks ago." c."I am an artist and work with oil paints." d."I have a history of a cardiac dysrhythmia."

B: The client with GBS often relates a history of acute illness, trauma, surgery, or immunization 1 to 3 weeks before the onset of neurologic symptoms. The other statements do not correlate with GBS.

Which of the following describes decerebrate posturing?

Back arched; rigid extension of all four extremities. Decerebrate posturing occurs in patients with damage to the upper brain stem, midbrain, or pons and is demonstrated clinically by arching of the back, rigid extension of the extremities, pronation of the arms, and plantar flexion of the feet. Internal rotation and adduction of arms with flexion of the elbows, wrists, and fingers described decorticate posturing, which indicates damage to corticospinal tracts and cerebral hemispheres.

A 23-year-old client has been hit on the head with a baseball bat. The nurse notes clear fluid draining from his ears and nose. Which of the following nursing interventions should be done first?

Check the fluid for dextrose with a dipstick Clear fluid from the nose or ear can be determined to be cerebral spinal fluid or mucous by the presence of dextrose. Placing the client flat in bed may increase ICP and promote pulmonary aspiration. The nose wouldn't be suctioned because of the risk for suctioning brain tissue through the sinuses. Nothing is inserted into the ears or nose of a client with a skull fracture because of the risk of infection.

A client's spouse expresses concern that the client, who has Guillain-Barré syndrome (GBS), is becoming very depressed and will not leave the house. What is the nurse's best response?

Contact the Guillain-Barré Syndrome Foundation International for resources. Here is their contact information."

A client has signs of increased ICP. Which of the following is an early indicator of deterioration in the client's condition?

Decrease in LOC A decrease in the client's LOC is an early indicator of deterioration of the client's neurological status. Changes in LOC, such as restlessness and irritability, may be subtle. Widening of the pulse pressure, decrease in the pulse rate, and dilated, fixed pupils occur later if the increased ICP is not treated.

The client with a head injury has been urinating copious amounts of dilute urine through the Foley catheter. The client's urine output for the previous shift was 3000 ml. The nurse implements a new physician order to administer:

Desmopressin (DDAVP, stimate) A complication of a head injury is diabetes insipidus, which can occur with insult to the hypothalamus, the antidiuretic storage vesicles, or the posterior pituitary gland. Urine output that exceeds 9 L per day generally requires treatment with desmopressin. Dexamethasone, a glucocorticoid, is administered to treat cerebral edema. This medication may be ordered for the head injured patient. Ethacrynic acid and mannitol are diuretics, which would be contraindicated.

A 23-year-old patient with a recent history of encephalitis is admitted to the medical unit with new onset generalized tonic-clonic seizures. Which nursing activities included in the patient's care will be best to delegate to an LPN/LVN whom you are supervising?

Document the onset time, nature of seizure activity, and postictal behaviors for all seizures. Administration of medications is included in LPN education and scope of practice. Collection of data about the seizure activity may be accomplished by an LPN/LVN who observes initial seizure activity. An LPN/LVN would know to call the supervising RN immediately if a patient started to seize. Documentation of the seizure, patient teaching, and planning of care are complex activities that require RN level education and scope of practice.

A client with head trauma develops a urine output of 300 ml/hr, dry skin, and dry mucous membranes. Which of the following nursing interventions is the most appropriate to perform initially?

Evaluate urine specific gravity Urine output of 300 ml/hr may indicate diabetes insipidus, which is a failure of the pituitary to produce anti-diuretic hormone. This may occur with increased intracranial pressure and head trauma; the nurse evaluates for low urine specific gravity, increased serum osmolarity, and dehydration. There's no evidence that the client is experiencing renal failure. Providing emollients to prevent skin breakdown is important, but doesn't need to be performed immediately. Slowing the rate of IV fluid would contribute to dehydration when polyuria is present.

A client who had a transsphenoidal hypophysectomy should be watched carefully for hemorrhage, which may be shown by which of the following signs?

Frequent swallowing Frequent swallowing after brain surgery may indicate fluid or blood leaking from the sinuses into the oropharynx. Blood or fluid draining from the ear may indicate a basilar skull fracture

The nurse is caring for a client who suffered a spinal cord injury 48 hours ago. The nurse monitors for GI complications by assessing for:

Hyperactive bowel sounds After spinal cord injury, the client can develop paralytic ileus, which is characterized by the absence of bowel sounds and abdominal distention. Development of a stress ulcer can be detected by hematest positive NG tube aspirate or stool. A history of diarrhea is irrelevant.

A client with C7 quadriplegia is flushed and anxious and complains of a pounding headache. Which of the following symptoms would also be anticipated?

Hypertension and bradycardia Hypertension, bradycardia, anxiety, blurred vision, and flushing above the lesion occur with autonomic dysreflexia due to uninhibited sympathetic nervous system discharge. The other options are incorrect.

A client comes into the ER after hitting his head in an MVA. He's alert and oriented. Which of the following nursing interventions should be done first?

Immobilize the client's head and neck All clients with a head injury are treated as if a cervical spine injury is present until x-rays confirm their absence. ROM would be contraindicated at this time. There is no indication that the client needs a chest x-ray. The airway doesn't need to be opened since the client appears alert and not in respiratory distress. In addition, the head-tilt chin-lift maneuver wouldn't be used until the cervical spine injury is ruled out.

The nurse is evaluating neurological signs of the male client in spinal shock following spinal cord injury. Which of the following observations by the nurse indicates that spinal shock persists?

Inability to elicit a Babinski's reflex Resolution of spinal shock is occurring when there is a return of reflexes (especially flexors to noxious cutaneous stimuli), a state of hyperreflexia rather than flaccidity, reflex emptying of the bladder, and a positive Babinski's reflex.

The nurse is planning care for the client in spinal shock. Which of the following actions would be least helpful in minimizing the effects of vasodilation below the level of the injury?

Moving the client quickly as one unit Reflex vasodilation below the level of the spinal cord injury places the client at risk for orthostatic hypotension, which may be profound. Measures to minimize this include measuring vital signs before and during position changes, use of a tilt-table with early mobilization, and changing the client's position slowly. Venous pooling can be reduced by using Teds (compression stockings) or pneumatic boots. Vasopressor medications are administered per protocol.

A nurse assesses a client who has episodes of autonomic dysreflexia. Which of the following conditions can cause autonomic dysreflexia?

Noxious stimuli Noxious stimuli, such as a full bladder, fecal impaction, or a decub ulcer, may cause autonomic dysreflexia. A headache is a symptom of autonomic dysreflexia, not a cause. Autonomic dysreflexia is most commonly seen with injuries at T10 or above. Neurogenic shock isn't a cause of dysreflexia.

The nurse is caring for a client admitted with spinal cord injury. The nurse minimizes the risk of compounding the injury most effectively by:

Placing the client on a Stryker frame Spinal immobilization is necessary after spinal cord injury to prevent further damage and insult to the spinal cord. Whenever possible, the client is placed on a Stryker frame, which allows the nurse to turn the client to prevent complications of immobility, while maintaining alignment of the spine. If a Stryker frame is not available, a firm mattress with a bed board should be used.

A 30-year-old was admitted to the progressive care unit with a C5 fracture from a motorcycle accident. Which of the following assessments would take priority?

Pulse ox readings After a spinal cord injury, ascending cord edema may cause a higher level of injury. The diaphragm is innervated at the level of C4, so assessment of adequate oxygenation and ventilation is necessary. Although the other options would be necessary at a later time, observation for respiratory failure is the priority.

After falling 20', a 36-year-old man sustains a C6 fracture with spinal cord transaction. Which other findings should the nurse expect?

Quadriplegia with gross arm movement and diaphragmatic breathing A client with a spinal cord injury at levels C5 to C6 has quadriplegia with gross arm movement and diaphragmatic breathing. Injury levels C1 to C4 leads to quadriplegia with total loss of respiratory function. Paraplegia with intercostal muscle loss occurs with injuries at T1 to L2. Injuries below L2 cause paraplegia and loss of bowel and bladder control.

The 25-year-old client with an SCI is sharing with the nurse that he is worried about how his family will be able to survive financially until he can go back to work. Which intervention should the nurse implement?

Refer the client to the social worker about applying for disability. he social worker is responsible for assisting the client with financial concerns. The ASIA assists clients to live with their SCI, and the rehabilitation commission can assist with employment.

An 18-year-old client is admitted with a closed head injury sustained in a MVA. His intracranial pressure (ICP) shows an upward trend. Which intervention should the nurse perform first?

Reposition the client to avoid neck flexion The nurse should first attempt nursing interventions, such as repositioning the client to avoid neck flexion, which increases venous return and lowers ICP. If nursing measures prove ineffective, notify the physician, who may prescribe mannitol, pentobarbital, or hyperventilation therapy.

Which of the following interventions describes an appropriate bladder program for a client in rehabilitation for spinal cord injury?

Schedule intermittent catheterization every 2 to 4 hours Intermittent catherization should begin every 2 to 4 hours early in the treatment. When residual volume is less than 400 ml, the schedule may advance to every 4 to 6 hours. Indwelling catheters may predispose the client to infection and are removed as soon as possible. Crede's maneuver is not used on people with spinal cord injury.

Which of the following respiratory patterns indicate increasing ICP in the brain stem?

Slow, irregular respirations Neural control of respiration takes place in the brain stem. Deterioration and pressure produce irregular respiratory patterns. Rapid, shallow respirations, asymmetric chest movements, and nasal flaring are more characteristic of respiratory distress or hypoxia.

Which of the following conditions indicates that spinal shock is resolving in a client with C7 quadriplegia?

Spasticity Spasticity, the return of reflexes, is a sign of resolving shock. Spinal or neurogenic shock is characterized by hypotension, bradycardia, dry skin, flaccid paralysis, or the absence of reflexes below the level of injury. The absence of pain sensation in the chest doesn't apply to spinal shock. Spinal shock descends from the injury, and respiratory difficulties occur at C4 and above.

A client with a C6 spinal injury would most likely have which of the following symptoms?

Tetraplegia Tetraplegia occurs as a result of cervical spine injuries. Paraplegia occurs as a result of injury to the thoracic cord and below.

The nurse and a UAP are caring for a client with right-sided paralysis. Which action by the UAP requires the nurse to intervene?

The UAP places the gait belt under the client's axilla prior to ambulating. The gait belt should be around the waist because this is the client's center of gravity. All other options are appropriate interventions for the UAP and would not require intervention.

A client has a cervical spine injury at the level of C5. Which of the following conditions would the nurse anticipate during the acute phase?

The need for mechanical ventilation The diaphragm is stimulated by nerves at the level of C4. Initially, this client may need mechanical ventilation due to cord edema. This may resolve in time. Absent corneal reflexes, decerebrate posturing, and hemiplegia occur with brain injuries, not spinal cord injuries.

A client with a cervical spine injury has Gardner-Wells tongs inserted for which of the following reasons?

To immobilize the cervical spine Gardner-Wells, Vinke, and Crutchfield tongs immobilize the spine until surgical stabilization is accomplished.

After a hypophysectomy, vasopressin is given IM for which of the following reasons?

To replace antidiuretic hormone (ADH) normally secreted by the pituitary. After hypophysectomy, or removal of the pituitary gland, the body can't synthesize ADH. Somatropin or growth hormone, not vasopressin is used to treat growth failure. SIADH results from excessive ADH secretion. Mannitol or corticosteroids are used to decrease cerebral edema.

A client is at risk for increased ICP. Which of the following would be a priority for the nurse to monitor?

Unequal pupil size Increasing ICP causes unequal pupils as a result of pressure on the third cranial nerve. Increasing ICP causes an increase in the systolic pressure, which reflects the additional pressure needed to perfuse the brain. It increases the pressure on the vagus nerve, which produces bradycardia, and it causes an increase in body temperature from hypothalamic damage.

A client who is regaining consciousness after a craniotomy becomes restless and attempts to pull out her IV line. Which nursing intervention protects the client without increasing her ICP?

Wrap her hands in soft "mitten" restraints It is best for the client to wear mitts which help prevent the client from pulling on the IV without causing additional agitation. Using a jacket or wrist restraint or tucking the client's arms and hands under the draw sheet restrict movement and add to feelings of being confined, all of which would increase her agitation and increase ICP.

The diagnosis of the fracture is confirmed and T.W. is transferred from the ED to the surgical intensive care unit (SICU). T.W. Is taken for surgery 48 hours after the accident for spinal stabilization. He spends 2 additional days in SICU and 5 days in the neurology unit and now is in the rehabilitation unit. He continues to have paralysis of this lower extremities. Shortly after the transfer T.W. turns his call light and asks for medication for headache. As you enter the room, you immediately note that T.W.'s face is flushed and he is profusely sweating. What complications do you suspect?

autonomic dysreflexia

The nurse is caring for a patient treated with alteplase following a stroke. What assessment finding is the highest priority for the nurse? a. Client's blood pressure is 144/90. b. Client is having epistaxis. c. Client ate only half of the last meal. d. Client continues to be drowsy.

b. Alteplase can cause bleeding, epistaxis is a nosebleed and there fore a concern. BP of 144/90 is acceptable for a stroke patient.

The client diagnosed with lung cancer has developed metastasis to the brain. Which problem would be priority for this client? l 1. Anticipatory grieving. l 2. Impaired gas exchange. l 3. Altered nutritional status. l 4. Alteration in comfort.

correct answer 1: Anticipatory grieving is priority because brain metastasis is a terminal diagnosis, indicating death within 6 months or less. With the development of brain metastasis, the nurse must address death and dying issues, which is why this is priority over all the other client problems.

Which neurotransmitter is responsible for may of the functions of the frontal lobe?

dopamine The frontal lobe primarily functions to regulate thinking, planning, and affect. Dopamine is known to circulate widely throughout this lobe, which is why it's such an important neurotransmitter in schizophrenia.

A 40-year-old paraplegic must perform intermittent catheterization of the bladder. Which of the following instructions should be given?

"Clean the meatus with soap and water." Intermittent catheterization may be performed chronically with clean technique, using soap and water to clean the urinary meatus. The meatus is always cleaned from front to back in a woman, or in expanding circles working outward from the meatus in a man. It isn't necessary to measure the urine. The catheter doesn't need to be rotated during removal.

The 29-year-old client who was employed as a forklift operator sustains a traumatic brain injury secondary to a motor-vehicle accident. The client is being discharged from the rehabilitation unit after three (3) months and has cognitive deficits. Which goal would be most realistic for this client? 1. The client will return to work within six (6) months. 2. The client is able to focus and stay on task for 10 minutes. 3. The client will be able to dress self without assistance. 4. The client will regain bowel and bladder control.

"Cognitive" pertains to mental processes of comprehension, judgment, memory, and reasoning. Therefore, an appropriate goal would be for the client to stay on task for 10 minutes.

What factors are suspected to be associated with the development of Guillain-Barré syndrome (GBS)? Select all that apply.

- Campylobacter jejune - Influenza - Epstein-barr Rationale:GBS is associated with bacterial infection, especially infection with Campylobacter jejuni. Influenza, Epstein-Barr, and cytomegalovirus viral infections have also been associated with GBS.

Which change in the cerebrospinal fluid (CSF) indicates to the nurse that a patient may have bacterial meningitis?

- Cloudy, turbid CSF Rationale:Clear fluid is a sign of viral meningitis. Increased white blood cells, increased protein, and decreased glucose are signs of bacterial meningitis.

A patient has Guillain-Barré syndrome. Which interdisciplinary health care team members does the nurse plan to collaborate with to help prevent pressure ulcers related to immobility in this patient? Select all that apply.

- Family members - Dietician - Occupation therapist (OT)

Which is the most effective way for a college student to minimize the risk for bacterial meningitis?

- Get the meningococcal vaccine Rationale: Individuals aged 16-21 years have the highest rates of meningococcal infection and should be immunized against the virus. Adults are advised to get an initial or booster vaccine if living in a shared residence (residence hall, military barracks, group home), traveling or residing in countries in which the disease is common, or are immunocompromised due to a damaged or surgically removed spleen or a serum complement deficiency. Avoiding large crowds is helpful, but is not practical for a college student. Taking a daily vitamin is helpful, but is not the best way to safeguard against bacterial meningitis. Taking prophylactic antibiotics is inappropriate because it leads to antibiotic-resistant strains of microorganisms.

The patient presents with complaints of pain, numbness, and paresthesias for the past two days and now has sudden onset of lower extremity muscle weakness. The nurse expects the patient to be tested for which neurological disease?

- Guillan Barre Syndrome Rationale:Pain, numbness, parasthesias, and sudden onset of muscle weakness are classic signs of Guillain-Barré Syndrome. Symptoms of Bell's palsy include facial paralysis; the face appears masklike and sags. Myasthenia gravis is characterized by diplopia and dysphagia. Trigeminal neuralgia is a chronic pain syndrome; this patient had symptoms that came on suddenly.

Which laboratory result is consistent with the diagnosis of Guillain-Barré syndrome (GBS)?

- Increased protein in CSF Rationale:An increase in the cerebrospinal fluid protein level occurs in GBS due to the release of plasma proteins from inflammation and damage to nerve roots. This release does not affect serum protein levels, which remain normal. The rheumatoid factor is not elevated in this disease and the erythrocyte sedimentation rate could rise.

Which statement best describes the pathophysiologic changes associated with Guillain-Barré syndrome (GBS)?

- It is an immune disorder in which the immune system destroys the myelin sheath of peripheral nerves Rationale:In GBS, the immune system destroys the myelin sheath, causing demyelination of the nerves leading to weakness and sensory abnormalities. Myasthenia gravis is the autoimmune disorder resulting in decreased acetylcholine. GBS is not an infectious disorder; however, it is often preceded by an infectious illness. Rheumatoid arthritis is the degenerative disease that leads to bone loss and muscle wasting.

What is the priority nursing intervention for a patient with rapidly ascending Guillain-Barré Syndrome (GBS)?

- Maintain airway patency and adequate gas exchange. Rationale:Inability to maintain an airway is a high risk and potentially fatal consequence of rapidly ascending GBS. The priority nursing intervention is to promote airway patency and adequate gas exchange. Plasmapheresis or IVIG are the primary treatment for GBS but the priority nursing intervention is to ensure a patent airway. Corticosteroids are not used unless medically indicated for associated diseases. Assessment and documentation of fluid status and vital signs is important but is not the priority intervention.

What is the priority assessment for a patient with bacterial meningitis?

- Neurological status

What other term is used to refer to Guillain-Barré syndrome?

- Polyradiculoneuropathy Rationale:Guillain-Barré syndrome is also called polyradiculoneuropathy because it is a polyneuropathy affecting peripheral nerves. Cranial polyneuritis is the term for Bell's palsy. Eaton-Lambert syndrome is a type of myasthenia gravis. Trigeminal neuralgia is a disease affecting the trigeminal nerve or cranial nerve V.

The nurse is caring for a patient with Guillain-Barré syndrome. Assessment findings include: Vital capacity- 14 mL/kg, respiratory rate- 30 breaths per minute, SpO2- 88%, rhonchi on lung auscultation and cyanosis. What is the best action for the nurse to take?

- Prepare for endotracheal intubation Rationale:Guillain-Barré syndrome is a neurological disorder involving the demyelination of nerves, including the nerves that help in respiration. As a result, the patient may be unable to breathe, manifested by a low vital capacity, tachypnea, low oxygen saturation, and inability to cough out secretions, leading to rhonchi on lung auscultation and cyanosis. This is a life-threatening condition and the patient should be intubated immediately to prevent fatal consequences. Oxygen therapy using a nasal cannula would be beneficial for patients experiencing mild difficulty with breathing. A respiratory therapist may help drain the secretions but may not be able to improve the neurological deficits. Elevating the head of the bed to at least 45 degrees may help promote lung expansion, but it may not promote the paralyzed respiratory muscles into action.

A patient is admitted with an exacerbation of Guillain-Barré syndrome (GBS), presenting with dyspnea. Which intervention does the nurse perform first?

- Raise the head of the bed to 45 degrees Rationale:The head of the patient's bed should be raised to 45 degrees because this allows for increased lung expansion, which improves the patient's ability to breathe. Intubation is indicated only if dyspnea is severe or oxygenation saturation does not respond to oxygen therapy. Close monitoring of respiratory status is indicated because of the acute stages of GBS. Instructing the patient on how to cough effectively is not the priority in this case. The patient should be suctioned only if needed to avoid vagal stimulation.

Which phase of Guillain-Barré syndrome (GBS) coincides with axonal regeneration?

- Recovery PhaseRationale:The recovery phase (gradually over 4 to 6 months, maybe up to 2 years) is thought to coincide with remyelination and axonal regeneration. The acute phase begins with onset of the first symptoms and ends when no further deterioration occurs. The plateau period follows the acute phase before remyelination and axonal regeneration occurs. Some patients do not completely recover and have permanent neurologic deficits, referred to as chronic GBS.

The nurse encourages a ventilated patient with advanced Guillain-Barré syndrome (GBS) to communicate by which simple technique?

- blinking yes or no

A client has been pronounced brain dead. Which findings would the nurse assess? Check all that apply.

-Dilated nonreactive pupils -Absent corneal reflex -Dilated nonreactive pupils A client who is brain dead typically demonstrates nonreactive dilated pupils and nonreactive or absent corneal and gag reflexes. The client may still have spinal reflexes such as deep tendon and Babinski reflexes in brain death. Decerebrate or decorticate posturing would not be seen.

Which client would the nurse identify as being most at risk for experiencing a CVA? 1. A 55-year-old African American male. 2. An 84-year-old Japanese female. 3. A 67-year-old Caucasian male. 4. A 39-year-old pregnant female.

1 African Americans have twice the rate of CVAs as Caucasians and men have a higher incidence than women; African Americans suffer more extensive damage from a CVA than do people of other cul- tural group

The client diagnosed with a mild concussion is being discharged from the emergency department. Which discharge instruction should the nurse teach the client's significant other? 1. Awaken the client every two (2) hours. 2. Monitor for increased intracranial pressure. 3. Observe frequently for hypervigilance. 4. Offer the client food every three (3) to four (4) hours.

1 Awakening the client every two(2) hours allows the identification of headache, dizziness, lethargy, irritabil- ity, and anxiety—all signs of postcon- cussion syndrome—that would warrant the significant other's taking the client back to the emergency department.

The client diagnosed with atrial fibrillation has experienced a transient ischemic attack (TIA). Which medication would the nurse anticipate being ordered for the client on discharge? 1. An oral anticoagulant medication. 2. A beta blocker medication. 3. An anti-hyperuricemic medication. 4. A thrombolytic medication.

1 The nurse would anticipate an oral anticoagulant, warfarin (Coumadin), to be prescribed to help prevent thrombi formation in the atria secondary to atrial fibrillation. The thrombi can become embolic and may cause a TIA or CVA (stroke).

The home health nurse is caring for a 28-year-old client with a T10 SCI who says, "I can't do anything. Why am I so worthless?" Which statement by the nurse would be the most therapeutic? 1. "This must be very hard for you. You're feeling worthless?" 2. "You shouldn't feel worthless—you are still alive." 3. "Why do you feel worthless? You still have the use of your arms." 4. "If you attended a work rehab program you wouldn't feel worthless."

1 Therapeutic communication addresses the client's feelings and attempts to allow the client to verbalize feelings; the nurse should be a therapeutic listener.

In assessing a client with a T12 SCI, which clinical manifestations would the nurse expect to find to support the diagnosis of spinal shock? 1. No reflex activity below the waist. 2. Inability to move upper extremities. 3. Complaints of a pounding headache. 4. Hypotension and bradycardia.

1 Spinal shock associated with SCI represents a sudden depression of reflex activity below the level of the injury. T12 is just above the waist; therefore, no reflex activity below the waist would be expected.

The nurse on the rehabilitation unit is caring for the following clients. Which client should the nurse assess first after receiving the change-of-shift report? 1. The client with a C6 SCI who is complaining of dyspnea and has crackles in the lungs. 2. . The client with an L4 SCI who is crying and very upset about being discharged home. 3. The client with an L2 SCI who is complaining of a headache and feeling very hot. 4. The client with a T4 SCI who is unable to move the lower extremities.

1 This client has signs/symptoms of a respiratory complication and should be assessed first.

The client diagnosed with a right-sided cerebrovascular accident is admitted to the rehabilitation unit. Which interventions should be included in the nursing care plan? Select all that apply. 1. Position the client to prevent shoulder adduction. 2. Turn and reposition the client every shift. 3. Encourage the client to move the affected side. 4. Perform quadriceps exercises three (3) times a day. 5. Instruct the client to hold the fingers in a fist.

1 & 3 Placing a small pillow under the shoulder will prevent the shoulder from adducting toward the chest and developing a contracture. The client should not ignore the paralyzed side, and the nurse must encourage the client to move it as much as possible; a written schedule may assist the client in exercising.

The client diagnosed with Guillain-Barré syndrome is admitted to the rehabilitation unit after 23 days in the acute care hospital. Which interventions should the nurse implement? Select all that apply. 1. Refer client to the physical therapist. 2. Include the speech therapist in the team. 3. Request a social worker consult. 4. Implement a regimen to address pain control. 5. Refer the client to the Guillain-Barré Syndrome Foundation.

1 . 3. 4. 5.

The nurse is preparing a client diagnosed with rule-out meningitis for a lumbar puncture. Which interventions should the nurse implement? Select all that apply. 1. Obtain an informed consent from the client or significant other. 2. Have the client empty the bladder prior to the procedure. 3. Place the client in a side-lying position with the back arched. 4. Instruct the client to breathe rapidly and deeply during the procedure. 5. Explain to the client what to expect during the procedure.

1, 2, 3, 5,

The client is being discharged following a transsphenoidal hypophysectomy. Which discharge instructions should the nurse teach the client? Select all that apply. 1. Sleep with the head of the bed elevated. 2. Keep a humidifier in the room. 3. Use caution when performing oral care. 4. Stay on a full liquid diet until seen by the HCP. 5. Notify the HCP if developing a cold or fever.

1, 2, 3, 5, 1. The client should sleep with the head of the bed elevated to promote drainage of the cerebrospinalfluid. 2. Humidified air will prevent drying of the nasal passages. 3. Because the incision for this surgery is just above the gumline, the client should not brush the front teeth. Oral care should be performed using a sponge until the incision has healed. 5. The HCP should be notified if the client develops an infection of any kind. A cold with sinus involvement and sneezing places the client at risk for opening the incision and developing a brain infection.

The nurse in the neurointensive care unit is caring for a client with a new C6 SCI who is breathing independently. Which nursing interventions should be implemented? Select all that apply. 1. Monitor the pulse oximetry reading. 2. Provide pureed foods six (6) times a day. 3. Encourage coughing and deep breathing. 4. Assess for autonomic dysreflexia. 5. Administer intravenous corticosteroids.

1, 3, 5, Oxygen is administered initially to prevent hypoxemia, which can worsen the spinal cord injury; therefore, the nurse should determine how much oxygen is reaching the periphery. Breathing exercises are supervised by the nurse to increase the strength and endurance of inspiratory muscles, especially those of the diaphragm. Corticosteroids are administered to decrease inflammation, which will decrease edema, and help prevent edema from ascending up the spinal cord, causing breathing difficulties.

The nurse is caring for a client diagnosed with meningitis. Which collaborative intervention should be included in the plan of care? 1. Administer antibiotics. 2. Obtain a sputum culture. 3. Monitor the pulse oximeter. 4. Assess intake and output.

1. A nurse administering antibiotics is a collaborative intervention because the HCP must write an order for the intervention; nurses cannot prescribe medications unless they have additional education and licensure and are nurse practitioners with prescriptive authority.

The client diagnosed with Guillain-Barré syndrome asks the nurse, "Will I ever get back to normal? I am so tired of being sick." Which statement is the best response by the nurse? 1. "You should make a full recovery within a few months to a year." 2. "Most clients with this syndrome have some type of residual disability." 3. "This is something you should discuss with the health-care team." 4. "The rehabilitation is short and you should be fully recovered within a month."

1. Clients with this syndrome usually have a full recovery, but it may take up to one (1) year.

The client diagnosed with a gunshot wound to the head assumes decorticate posturing when the nurse applies painful stimuli. Which assessment data obtained three (3) hours later would indicate the client is improving? 1. Purposeless movement in response to painful stimuli. 2. Flaccid paralysis in all four extremities. 3. Decerebrate posturing when painful stimuli are applied. 4. Pupils that are 6 mm in size and nonreactive on painful stimuli.

1. Purposeless movement indicates that the client's cerebral edema is decreasing. The best motor response is purposeful movement, but purposeless movement indicates an improvement over decorticate movement, which, in turn, is an improvement over decerebrate movement or flaccidity.

The client is diagnosed with a closed head injury and is in a coma. The nurse writes the client problem as "high risk for immobility complications." Which intervention would be included in the plan of care? 1. Position the client with the head of the bed elevated at intervals. 2. Perform active range-of-motion exercises every four (4) hours. 3. Turn the client every shift and massage bony prominences. 4. Explain all procedures to the client before performing them.

1. The head of the client's bed should be elevated to help the lungs expand and prevent stasis of secretions that could lead to pneumonia, a complication of immobility.

The public health nurse is giving a lecture on potential outbreaks of infectious meningitis. Which population is most at risk for an outbreak? 1. Clients recently discharged from the hospital. 2. Residents of a college dormitory. 3. Individuals who visit a third world country. 4. Employees in a high-rise office building.

2 Outbreaks of infectious meningitis are most likely to occur in dense community groups such as college campuses, jails, and military installations.

The client with a cervical fracture is being discharged in a halo device. Which teaching instruction should the nurse discuss with the client? 1. Discuss how to correctly remove the insertion pins. 2. Instruct the client to report reddened or irritated skin areas. 3. Inform the client that the vest liner cannot be changed. 4. Encourage the client to remain in the recliner as much as possible.

2 Reddened areas, especially under the brace, must be reported to the HCP because pressure ulcers can occur when wearing this appliance for an extended period.

The male client diagnosed with a brain tumor is scheduled for a magnetic resonance imaging (MRI) scan in the morning. The client tells the nurse that he is scared. Which response by the nurse indicates an appropriate therapeutic response? 1. "MRIs are loud but there will not be any invasive procedure done." 2. "You're scared. Tell me about what is scaring you." 3. "This is the least thing to be scared about—there will be worse." 4. "I can call the MRI tech to come and talk to you about the scan."

2 This is restating and offering self. Both are therapeutic responses.

The nurse is assessing a client experiencing motor loss as a result of a left-sided cere- brovascular accident (CVA). Which clinical manifestations would the nurse document? 1. Hemiparesis of the client's left arm and apraxia. 2. Paralysis of the right side of the body and ataxia. 3. Homonymous hemianopsia and diplopia. 4. Impulsive behavior and hostility toward family.

2 the most common motor dysfunction of a CVA is paralysis of one side of the body, hemiplegia; in this case with a left-sided CVA, the paralysis would affect the right side. Ataxia is an impaired ability to coordinate movement.

The 34-year-old male client with an SCI is sharing with the nurse that he is worried about finding employment after being discharged from the rehabilitation unit. Which intervention should the nurse implement? 1. Refer the client to the American Spinal Cord Injury Association (ASIA). 2. Refer the client to the state rehabilitation commission .3. Ask the social worker about applying for disability.4 . Suggest that the client talk with his significant other about this concern.

2 The rehabilitation commission of each state will help evaluate and determine if the client can receive training or education for another occupation after injur

Which statement by the client supports the diagnosis of Guillain-Barré syndrome? 1. "I just returned from a short trip to Japan." 2. "I had a really bad cold just a few weeks ago." 3. "I think one of the people I work with had this." 4. "I have been taking some herbs for more than a year."

2 This syndrome is usually preceded by a respiratory or gastrointestinal infection one (1) to four (4) weeks prior to the onset of neurological deficits.

The client diagnosed with Guillain-Barré syndrome is on a ventilator. Which intervention will assist the client to communicate with the nursing staff? 1. Provide an erase slate board for the client to write on. 2. Instruct the client to blink once for "no" and twice for "yes." 3. Refer to a speech therapist to help with communication. 4. Leave the call light within easy reach of the client.

2 - The client will not be able to use the arms as a result of the paralysis but can blink the eyes as long as the nurse asks simple "yes-or-no" questions.

The nurse caring for the client diagnosed with Guillain-Barré syndrome writes the client problem "impaired physical mobility." Which long-term goal should be written for this problem? 1. The client will have no skin irritation. 2. The client will have no muscle atrophy. 3. The client will perform range-of-motion exercises. 4. The client will turn every two (2) hours while awake.

2 - The client with Guillain-Barré syndrome will not be able to move the extremities; therefore, preventing muscle atrophy is an appropriate long-term goal.

The nurse is caring for a client diagnosed with an epidural hematoma. Which nursing interventions should the nurse implement? Select all that apply. 1. Maintain the head of the bed at 60 degrees of elevation. 2. Administer stool softeners daily. 3. Ensure that pulse oximeter reading is higher than 93%. 4. Perform deep nasal suction every two (2) hours. 5. Administer mild sedatives.

2, 3, 5, Stool softeners are initiated to prevent the Valsalva maneuver, which increases intracranial pressure. Oxygen saturation higher than 93% ensures oxygenation of the brain tissues; decreasing oxygen levels increase cerebral edema Mild sedatives will reduce the client's agitation; strong narcotics would not be administered because they decrease the client's level of consciousness

The nurse is developing a plan of care for a client diagnosed with aseptic meningitis secondary to a brain tumor. Which nursing goal would be most appropriate for the client problem "altered cerebral tissue perfusion"? 1. The client will be able to complete activities of daily living. 2. The client will be protected from injury if seizure activity occurs. 3. The client will be afebrile for 48 hours prior to discharge. 4. The client will have elastic tissue turgor with ready recoil.

2, A client with a problem of altered cerebral tissue perfusion is at risk for seizure activity secondary to focal areas of cortical irritability; therefore, the client should be on seizure precautions.

The resident in a long-term care facility fell during the previous shift and has a lacer- ation in the occipital area that has been closed with Steri-Strips. Which signs/ symptoms would warrant transferring the resident to the emergency department? 1. A 4-cm area of bright red drainage on the dressing. 2. A weak pulse, shallow respirations, and cool pale skin. 3. Pupils that are equal, react to light, and accommodate. 4. Complaints of a headache that resolves with medication.

2. These signs/symptoms—weak pulse, shallow respirations, cool pale skin— indicate increased intracranial pressure from cerebral edema secondary to the fall, and they require immediate attention.

The client diagnosed with a closed head injury is admitted to the rehabilitation department. Which medication order would the nurse question? 1. A subcutaneous anticoagulant. 2. An intravenous osmotic diuretic. 3. An oral anticonvulsant. 4. An oral proton pump inhibitor.

2. An osmotic diuretic would be ordered in the acute phase to help decrease cerebral edema, but this medication would not be expected to be orderedin a rehabilitation unit.

The client is diagnosed with meningococcal meningitis. Which preventive measure would the nurse expect the health-care provider to order for the significant others in the home? 1. The Haemophilus influenzae vaccine. 2. Antimicrobial chemoprophylaxis. 3. A 10-day dose pack of corticosteroids. 4. A gamma globulin injection.

2. Chemoprophylaxis includes administering medication that will prevent infection or eradicate the bacteria and the development of symptoms in people who have been in close proximity to the client. Medications include rifampin (Rifadin), ciprofloxacin (Cipro), and ceftriaxone (Rocephin). (not 1 b/c vaccine must be administered prior to exposure to build up an immunity

Which statement best describes the scientific rationale for alternating a nonnarcotic antipyretic and a nonsteroidal anti-inflammatory drug (NSAID) every two (2) hours to a female client diagnosed with bacterial meningitis? 1. This regimen helps to decrease the purulent exudate surrounding the meninges. 2. These medications will decrease intracranial pressure and brain metabolism. 3. These medications will increase the client's memory and orientation. 4. This will help prevent a yeast infection secondary to antibiotic therapy.

2. Fever increases cerebral metabolism and intracranial pressure. Therefore, measures are taken to reduce body temperature as soon as possible, and alternating Tylenol and Motrin would be appropriate.

The client is diagnosed with a metastatic brain tumor, and radiation therapy is scheduled. The client asks the nurse, "Why not try chemotherapy first? It has helped my other tumors." The nurse's response is based on which scientific rationale? 1. Chemotherapy is only used as a last resort in caring for clients with brain tumors. 2. The blood-brain barrier prevents medications from reaching the brain. 3. Radiation therapy will have fewer side effects than chemotherapy. 4. Metastatic tumors become resistant to chemotherapy and it becomes useless.

2. The blood-brain barrier is the body's defense mechanism for protecting the brain from chemical effects; in this case, it prevents the chemotherapy from being able to wor

The health-care provider scheduled a lumbar puncture for a client admitted with rule-out Guillain-Barré syndrome. Which preprocedure intervention has priority? 1. Keep the client NPO. 2. Instruct the client to void. 3. Place in the lithotomy position. 4. Assess the client's pedal pulse.

2. The client should void prior to this procedure to help prevent accidental puncture of the bladder during the procedure.

A 78-year-old client is admitted to the emergency department with numbness and weakness of the left arm and slurred speech. Which nursing intervention is priority? 1. Prepare to administer recombinant tissue plasminogen activator (rt-PA). 2. Discuss the precipitating factors that caused the symptoms. 3. Schedule for a STAT computed tomography (CT) scan of the head. 4. Notify the speech pathologist for an emergency consult.

3 A CT scan will determine if the client is having a stroke or has a brain tumor or another neurological disorder. If a CVA is diagnosed, the CT scan can determine if it is a hemorrhagic or ischemic accident and guide treatment.

The nurse is assessing the client diagnosed with bacterial meningitis. Which clinical manifestations would support the diagnosis of bacterial meningitis? 1. Positive Babinski's sign and peripheral paresthesia. 2. Negative Chvostek's sign and facial tingling. 3. Positive Kernig's sign and nuchal rigidity. 4. Negative Trousseau's sign and nystagmus.

3 A positive Kernig's sign (client unable to extend leg when lying flat) and nuchal rigidity (stiff neck) are signs of bacterial meningitis, occurring because the meninges surrounding the brain and spinal column are irritated.

The client is diagnosed with a pituitary tumor and is scheduled for a transsphenoidal hypophysectomy. Which preoperative instruction is important for the nurse to teach? 1. There will be a large turban dressing around the skull after surgery. 2. The client will not be able to eat for four (4) or five (5) days postop. 3. The client should not blow the nose for two (2) weeks after surgery. 4. The client will have to lie flat for 24 hours following the surgery.

3 Blowing the nose creates increased intracranial pressure and could result in a cerebrospinal fluid leak.

The client with a closed head injury has clear fluid draining from the nose. Which action should the nurse implement first? 1. Notify the health-care provider immediately. 2. Prepare to administer an antihistamine. 3. Test the drainage for presence of glucose. 4. Place 2 × 2 gauze under the nose to collect drainage.

3 The presence of glucose in drainage from the nose or ears indicates cerebrospinal fluid, and the HCP should be notified immediately once this is determined.

Which nursing task would be most appropriate for the nurse to delegate to the unlicensed assistive personnel? 1. Teach Credé's maneuver to the client needing to void. 2. Administer the tube feeding to the client who is quadriplegic. 3. Assist with bowel training by placing the client on the bedside commode. 4. Observe the client demonstrating self-catheterization technique.

3 The assistant can place the client on the bedside commode as part of bowel training; the nurse is responsible for the training but can delegate this task.

The client with a C6 SCI is admitted to the emergency department complaining of a severe pounding headache and has a BP of 180/110. Which intervention should the emergency department nurse implement? 1. Keep the client flat in bed. 2. Dim the lights in the room. 3. Assess for bladder distention. 4. Administer a narcotic analgesic.

3 This is an acute emergency causedby exaggerated autonomic responses to stimuli and only occurs afterspinal shock has resolved in the client with a spinal cord injury above T6. The most common cause is a full bladder.

The nurse has written a care plan for a client diagnosed with a brain tumor. Which is an important goal regarding self-care deficit? 1. The client will maintain body weight within two (2) pounds. 2. The client will execute an advance directive. 3. The client will be able to perform three (3) ADLs with assistance. 4. The client will verbalize feeling of loss by the end of the shift.

3 Performing activities of daily living is a goal for self-care deficit.

The client diagnosed with breast cancer has developed metastasis to the brain. Which prophylactic measure should the nurse implement? 1. Institute aspiration precautions. 2. Refer the client to Reach to Recovery. 3. Initiate seizure precautions. 4. Teach the client about mastectomy care.

3 The client diagnosed with metastatic lesions to the brain is at high risk for seizures.

The nurse is caring for the following clients. Which client would the nurse assess first after receiving the shift report? 1. The 22-year-old male client diagnosed with a concussion who is complaining someone is waking him up every two (2) hours. 2. The 36-year-old female client admitted with complaints of left-sided weakness who is scheduled for a magnetic resonance imaging (MRI) scan. 3. The 45-year-old client admitted with blunt trauma to the head after a motorcycle accident who has a Glasgow Coma Scale score of 6. 4. The 62-year-old client diagnosed with a cerebrovascular accident (CVA) who has expressive aphasia.

3. The Glasgow Coma Scale is used to determine a client's response to stimuli (eye-opening response, best verbal response, and best motor response) secondary to a neurological problem; scores range from 3 (deep coma) to 15 (intact neurological function). A client with a score of 6 should be assessed first by the nurse.

The client has sustained a severe closed head injury and the neurosurgeon is deter- mining if the client is "brain dead." Which data support that the client is brain dead? 1. When the client's head is turned to the right, the eyes turn to the right. 2. The electroencephalogram (EEG) has identifiable waveforms. 3. There is no eye activity when the cold caloric test is performed. 4. The client assumes decorticate posturing when painful stimuli are applied.

3. The cold caloric test, also called the oculovestibular test, is a test usedto determine if the brain is intact or dead. No eye activity indicates brain death. If the client's eyes moved, that would indicate that the brainstem is intact.

The intensive care nurse is caring for a client with a T1 SCI. When the nurse elevates the head of the bed 30 degrees, the client complains of lightheadedness and dizziness. The client's vital signs are T 99.2 ̊F, P 98, R 24, and BP 84/40. Which action should the nurse implement? 1. Notify the health-care provider ASAP. 2. Calm the client down by talking therapeutically. 3. Increase the IV rate by 50 mL/hour. 4. Lower the head of the bed immediately.

4 For the first two (2) weeks after an SCI above T7, the blood pressure tends to be unstable and low; slight elevations of the head of the bed can cause profound hypotension; therefore, the nurse should lower the head of the bed immediately.

The client has been diagnosed with a brain tumor. Which presenting signs and symptoms help to localize the tumor position? 1. Widening pulse pressure and bounding pulse. 2. Diplopia and decreased visual acuity. 3. Bradykinesia and scanning speech .4. Hemiparesis and personality changes.

4 Hemiparesis would localize a tumorto a motor area of the brain, and personality changes localize a tumor to the frontal lobe.

The client is diagnosed with an SCI and is scheduled for a magnetic resonance imaging (MRI) scan. Which question would be most appropriate for the nurse to ask prior to taking the client to the diagnostic test? 1. "Do you have trouble hearing?" 2. "Are you allergic to any type of dairy products?" 3. "Have you eaten anything in the last eight (8) hours?" 4. "Are you uncomfortable in closed spaces?"

4 MRI scans are often done in a very confined space; many people who have claustrophobia must be medicated or even rescheduled for the procedure in an open MRI machine, w

The 85-year-old client diagnosed with a stroke is complaining of a severe headache. Which intervention should the nurse implement first? 1. Administer a nonnarcotic analgesic 2. Prepare for STAT magnetic resonance imaging (MRI). 3. Start an intravenous infusion with D5W at 100 mL/hr. 4. Complete a neurological assessment.

4 The nurse must complete a neurological assessment to help determine the cause of the headache before taking any further action.

The client diagnosed with a brain tumor has a diminished gag response. Which intervention should the nurse implement? 1. Make the client NPO until seen by the health-care provider. 2. Position the client in low Fowler's position for all meals. 3. Place the client on a mechanically ground diet. 4. Teach the client to direct food and fluid toward the unaffected side.

4 to decrease the risk of aspiration, the client should direct food to the unaffected side of the throat; this helps the client to be able to use the side of the throat that is functioning.

Which assessment data should the nurse assess in the client diagnosed with Guillain-Barré syndrome? 1. An exaggerated startle reflex and memory changes. 2. Cogwheel rigidity and inability to initiate voluntary movement. 3. Sudden severe unilateral facial pain and inability to chew. 4. Progressive ascending paralysis of the lower extremities and numbness.

4. Ascending paralysis is the classic symptom of Guillain-Barré syndrome.

Which type of precautions should the nurse implement for the client diagnosed with septic meningitis? 1. Standard Precautions. 2. Airborne Precautions. 3. Contact Precautions. 4. Droplet Precautions.

4. Droplet Precautions are respiratory precautions used for organisms that have a limited span of transmission. Precautions include staying at least four (4) feet away from the client or wearing a standard isolation mask and gloves when coming in close contact with the client. Clients are in isolation for 24 to 48 hours after initiation of antibiotics

Which priority client problem should be included in the care plan for the client diagnosed with Guillain-Barré syndrome? 1. High risk for injury. 2. Fear and anxiety. 3. Altered nutrition. 4. Ineffective breathing pattern.

4. Guillain-Barré syndrome has ascending paralysis causing respiratory failure. Therefore, breathing pattern is priority.

The 29-year-old client is admitted to the medical floor diagnosed with meningitis. Which assessment by the nurse has priority? 1. Assess lung sounds. 2. Assess the six cardinal fields of gaze. 3. Assess apical pulse. 4. Assess level of consciousness.

4. Meningitis directly affects the client's brain. Therefore, assessing the neurological status would have priority for this client.

The client is admitted to the medical floor with a diagnosis of closed head injury. Which nursing intervention has priority? 1. Assess neurological status. 2. Monitor pulse, respiration, and blood pressure. 3. Initiate an intravenous access. 4. Maintain an adequate airway.

4. The most important nursing goal in the management of a client with a head injury is to establish and maintain an adequate airway.

The nurse is enjoying a day at the lake and witnesses a water skier hit the boat ramp. The water skier is in the water not responding to verbal stimuli. The nurse is the first health-care provider to respond to the accident. Which intervention should be implemented first? 1. Assess the client's level of consciousness. 2. Organize onlookers to remove the client from the lake. 3. Perform a head-to-toe assessment to determine injuries. 4. Stabilize the client's cervical spine.

4. The nurse should always assume that a client with traumatic head injury may have sustained spinal cord injury. Mov- ing the client could further injure the spinal cord and cause paralysis; there- fore, the nurse should stabilize the cer- vical spinal cord as best as possible prior to removing the client from the water.

The client diagnosed with Guillain-Barré syndrome is on a ventilator. When the wife comes to visit she starts crying uncontrollably, and the client starts fighting the ventilator because his wife is upset. Which action should the nurse implement? 1. Tell the wife she must stop crying. 2. Escort the wife out of the room. 3. Medicate the client immediately. 4. Acknowledge the wife's fears.

4. it is scary for a wife to see her loved one with a tube down his mouth and all the machines around them. The nurse should help the wife by acknowl- edging her fears.

A nursing student studying traumatic brain injuries (TBIs) should recognize which facts about these disorders? (Select all that apply.) a. A client with a moderate trauma may need hospitalization. b. A Glasgow Coma Scale score of 10 indicates a mild brain injury. c. Only open head injuries can cause a severe TBI. d. A client with a Glasgow Coma Scale score of 3 has severe TBI. e. The terms mild TBI and concussion have similar meanings.

A D E

The nurse is planning care for a client experiencing agnosia secondary to a cerebrovas- cular accident. Which collaborative intervention will be included in the plan of care? 1. Observe the client swallowing for possible aspiration. 2. Position the client in a semi-Fowler's position when sleeping. 3. Place a suction setup at the client's bedside during meals. 4. Refer the client to an occupational therapist for evaluation.

A collaborative intervention is an intervention in which another health-care discipline—in this case, occupational therapy—is used in the care of the client.

A nurse is providing community screening for risk factors associated with stroke. Which client would the nurse identify as being at highest risk for a stroke?a. A 27-year-old heavy cocaine user b. A 30-year-old who drinks a beer a day c. A 40-year-old who uses seasonal antihistamines d. A 65-year-old who is active and on no medications

ANS: A Heavy drug use, particularly cocaine, is a risk factor for stroke. Heavy alcohol use is also a risk factor, but one beer a day is not considered heavy drinking. Antihistamines may contain phenylpropanolamine, which also increases the risk for stroke, but this client uses them seasonally and there is no information that they are abused or used heavily. The 65-year-old has only age as a risk factor.

A client in the emergency department is having a stroke and needs a carotid artery angioplasty with stenting. The clients mental status is deteriorating. What action by the nurse is most appropriate? a. Attempt to find the family to sign a consent. b. Inform the provider that the procedure cannot occur. c. Nothing; no consent is needed in an emergency. d. Sign the consent form for the client.

ANS: A The nurse should attempt to find the family to give consent. If no family is present or can be found, under the principle of emergency consent, a life-saving procedure can be performed without formal consent. The nurse should not just sign the consent form.

A client is in the clinic for a follow-up visit after a moderate traumatic brain injury. The clients spouse is very frustrated, stating that the clients personality has changed and the situation is intolerable. What action by the nurse is best?a. Explain that personality changes are common following brain injuries. b. Ask the client why he or she is acting out and behaving differently. c. Refer the client and spouse to a head injury support group. d. Tell the spouse this is expected and he or she will have to learn to cope.

ANS: A Personality and behavior often change permanently after head injury. The nurse should explain this to the spouse. Asking the client about his or her behavior isnt useful because the client probably cannot help it. A referral might be a good idea, but the nurse needs to do something in addition to just referring the couple. Telling the spouse to learn to cope belittles the spouses concerns and feelings.

A client has a traumatic brain injury. The nurse assesses the following: pulse change from 82 to 60 beats/min, pulse pressure increase from 26 to 40 mm Hg, and respiratory irregularities. What action by the nurse takes priority?a. Call the provider or Rapid Response Team. b. Increase the rate of the IV fluid administration. c. Notify respiratory therapy for a breathing treatment. d. Prepare to give IV pain medication.

ANS: A These manifestations indicate Cushings syndrome, a potentially life-threatening increase in intracranial pressure (ICP), which is an emergency. Immediate medical attention is necessary, so the nurse notifies the provider or the Rapid Response Team. Increasing fluids would increase the ICP. The client does not need a breathing treatment or pain medication.

A client is admitted with a brain abscess. Which diagnostic assessment intervention does the nurse question as nonspecific to the diagnosis? a. Bone scan b. Sinus x-rays c. Throat culture d. Electroencephalogram (EEG)

ANS: A A bone scan is done to determine new areas of bone growth, areas of metastatic lesions, and osteoporosis. An EEG is obtained to localize the lesion. Throat, ear, nose, and blood (aerobic and anaerobic) cultures are done to determine the primary source of infection. Sinus and mastoid x-rays are requested to see if they are the primary source of the infection causing the brain abscess.

The nurse is assessing a client with a traumatic brain injury after a skateboarding accident. Which symptom is the nurse most concerned about? a. Asymmetric pupils b. Restlessness c. Head laceration d. Amnesia

ANS: A Asymmetric (uneven) pupils, loss of light reaction, or unilateral or bilateral dilated pupils are a sign of a severe traumatic brain injury. Pupil changes are treated as herniation of the brain from increased intracranial pressure (ICP) until proven differently. The nurse should report and document any changes in pupil size, shape, and reactivity to the health care provider immediately because they could indicate an increase in ICP. Amnesia, a head laceration, and restlessness can be symptoms of mild traumatic brain injuries.

A client with a traumatic brain injury from a motor vehicle crash is monitored for signs of increased intracranial pressure (ICP). Which sign does the nurse monitor for? a. Changes in breathing pattern b. Reactive pupils c. Dizziness d. Increasing level of consciousness

ANS: A Changes in breathing pattern may cause hypoxia and hypercapnia, which can increase ICP. Dizziness is indicative of brain injury. Increasing level of consciousness and reactive pupils are desired outcomes for this client.

The nurse is monitoring a postoperative craniotomy client with increased intracranial pressure (ICP). Which pharmacologic agent does the nurse expect to be requested to maintain the ICP within a specified range? a. Mannitol (Osmitrol) b. Hydrochlorothiazide (HydroDIURIL) c. Dexamethasone (Decadron) d. Phenytoin (Dilantin)

ANS: A Mannitol is an osmotic diuretic used specifically to treat cerebral edema. Glucocorticoids have no demonstrated benefit in reducing ICP. Hydrochlorothiazide is only a mild diuretic; a loop diuretic such as furosemide (Lasix) is commonly used along with mannitol to reduce ICP. Dilantin is used to treat seizure activity caused by increased ICP.

A client with a history of atrial fibrillation is receiving sodium heparin 24 hours after receiving thrombolytic therapy for a stroke. Which emergency drug does the nurse ensure is on the floor? a. Protamine sulfate b. Vitamin K c. Physostigmine d. Narcan

ANS: A Protamine sulfate is used to reverse the effects of heparin in case of adverse effects. Narcan (naloxone) is used to reverse the effects of a narcotic overdose. Vitamin K is used to reverse the effects of warfarin. Physostigmine is an acetylcholinesterase inhibitor used to treat myasthenia gravis.

A client in the emergency department (ED) has slurred speech, confusion, and visual problems, and has been having intermittent episodes of worsening symptoms. The symptoms have a gradual onset. The client also has a history of hypertension and atherosclerosis. What does the nurse suspect that the client is probably experiencing? a. Thrombotic stroke b. Transient ischemic attack c. Hemorrhagic stroke d. Embolic stroke

ANS: A The client's symptoms fit the description of a thrombotic stroke. Symptoms of embolic stroke have a sudden onset, unlike this client's symptoms. The client would be in a coma if a hemorrhagic stroke had occurred. Intermittent episodes of slurred speech, confusion, and visual problems are transient ischemic attacks, which often are warning signs of an impending ischemic stroke.

The nurse is monitoring a client after supratentorial surgery. Which sign does the nurse report immediately to the provider? a. Decorticate positioning b. Periorbital edema c. Moderate amount of serosanguineous drainage on the head dressing d. Bilateral ecchymoses of both eyes

ANS: A The major complications of supratentorial surgery are increased intracranial pressure from cerebral edema or hydrocephalus and hemorrhage. Decorticate positioning indicates damage to the pathway between the brain and the spinal cord. The client usually is rigid with flexion of arms, clenched fists, and extended rigid legs. Periorbital edema and a small-to-moderate amount of serosanguineous drainage are expected after a craniotomy. Ecchymoses in the facial region, especially around the eyes, are expected after a craniotomy.

A client has meningitis following brain surgery. What comfort measures may the nurse delegate to the unlicensed assistive personnel (UAP)? (Select all that apply.) a. Applying a cool washcloth to the head b. Assisting the client to a position of comfort c. Keeping voices soft and soothing d. Maintaining low lighting in the room e. Providing antipyretics for fever

ANS: A, B, C, D The client with meningitis often has high fever, pain, and some degree of confusion. Cool washcloths to the forehead are comforting and help with pain. Allowing the client to assume a position of comfort also helps manage pain. Keeping voices low and lights dimmed also helps convey caring in a nonthreatening manner. The nurse provides antipyretics for fever.

The nurse teaches a client with Guillain-Barré syndrome (GBS) about the recovery rate of this disorder. Which statement indicates that the client correctly understands the teaching? a."I need to see a lawyer because I do not expect to recover from this disease." b."I will have to take things slowly for several months after I leave the hospital." c."I expect to be able to return to work in construction soon after I get discharged." d."I wonder if my family will be able to manage my care now that I am paralyzed."

ANS: B Most clients make a full recovery from GBS. Recovery can take as long as 6 months to 2 years. Fatigue is a major lingering symptom for most of those diagnosed with this disorder. Clients are not permanently paralyzed. They are in an acute care environment during the acute phase of the disorder.

The nurse assesses a client with Guillain-Barré syndrome during plasmapheresis. Which complication does the nurse monitor for during this procedure?a.Tachycardiab.Hypovolemiac.Hyperkalemiad.Hemorrhage

ANS: B The client undergoing plasmapheresis is at risk for hypovolemia. The nurse monitors fluid status, assesses vital signs, and administers replacement fluid, as indicated. The other manifestations are not complications of plasmapheresis.

A client is eating a soft diet while recovering from a stroke. The client reports food accumulating in the cheek of the affected side. What is the nurse's best response? a. "Let's see if the dietitian can help." b. "Let's see if the speech-language pathologist can help." c. "Let's advance your diet to solid food." d. "Next time you eat, try lifting your chin when you swallow."

ANS: B The speech-language pathologist identifies strategies to prevent food from accumulating in the cheek of the affected side of a client recovering from a stroke. The correct technique to improve swallowing is the chin-tuck method; however, the speech pathologist will assist the client with tongue exercises that will help move the food bolus to the unaffected side. Solid food is not appropriate for the client with chewing and swallowing challenges. The dietitian consults with the health care team if the client has had weight loss problems, or if abnormal laboratory results indicate a nutritional deficit.

After a craniotomy, the nurse assesses the client and finds dry, sticky mucous membranes and restlessness. The client has IV fluids running at 75 mL/hr. What action by the nurse is best? a. Assess the clients magnesium level. b. Assess the clients sodium level. c. Increase the rate of the IV infusion. d. Provide oral care every hour.

ANS: B This client has manifestations of hypernatremia, which is a possible complication after craniotomy. The nurse should assess the clients serum sodium level. Magnesium level is not related. The nurse does not independently increase the rate of the IV infusion. Providing oral care is also a good option but does not take priority over assessing laboratory results.

A client is in the emergency department reporting a brief episode during which he was dizzy, unable to speak, and felt like his legs were very heavy. Currently the clients neurologic examination is normal. About what drug should the nurse plan to teach the client? a. Alteplase (Activase) b. Clopidogrel (Plavix) c. Heparin sodium d. Mannitol (Osmitrol)

ANS: B This clients manifestations are consistent with a transient ischemic attack, and the client would be prescribed aspirin or clopidogrel on discharge. Alteplase is used for ischemic stroke. Heparin and mannitol are not used for this condition

A client has had a traumatic brain injury and is mechanically ventilated. Which technique does the nurse use to prevent increasing intracranial pressure (ICP)? a. Suctioning the client frequently b. Maintaining PaCO2 levels at 35 mm Hg c. Assessing for Turner's sign d. Placing the client in the Trendelenburg position

ANS: B After the first 24 hours when a client is mechanically ventilated, keeping the PaCO2 levels at 35 mm Hg prevents vasodilation, which could increase ICP. CO2 is a powerful vasodilator. Turner's sign is a bluish gray discoloration in the flank region caused by acute pancreatitis. The head of the bed should be at 30 degrees; the Trendelenburg position will cause the client's ICP to increase. Although some suctioning is necessary, frequent suctioning should be avoided because it increases ICP.

A client is considering treatments for a malignant brain tumor. Which statement by the client indicates a need for further instruction by the nurse? a. "A combination of treatments might be necessary." b. "Antibiotics will help minimize the size of the tumor." c. "In a craniotomy, holes are cut in the skull to access the tumor." d. "The goal is to decrease tumor size and improve survival time."

ANS: B Antibiotics are used to treat a brain abscess, not a malignant tumor. Chemotherapy, radiation, and surgery are often used in conjunction with each other to treat malignancies. For a craniotomy, several burr holes are drilled into the skull, and a saw is used to remove a piece of bone (bone flap) to expose the tumor area. The goals of treatment of brain tumor are to decrease tumor size, improve quality of life, and improve survival time.

A client hospitalized for hypertension presses the call light and reports "feeling funny." When the nurse gets to the room, the client is slurring words and has right-sided weakness. What does the nurse do first? a. Calls the provider b. Assesses airway, breathing, and circulation c. Performs a neurologic check d. Assists the client to a sitting position

ANS: B The client must be evaluated within 10 minutes of having a stroke. The priority is assessment of the "ABCs"—airway, breathing, and circulation. Calling the Rapid Response Team, not the provider, after assessing ABCs would be appropriate. A neurologic check may be performed later, but is not the priority in this situation. A sitting position is used for hyperreflexia in the client with spinal cord injury to assist in lowering blood pressure.

A client has been admitted with a diagnosis of stroke (brain attack). The nurse suspects that the client has had a right hemisphere stroke because the client exhibits which symptoms? a. Aphasia and cautiousness b. Impulsiveness and smiling c. Quick to anger and frustration d. Inability to discriminate words

ANS: B bImpulsiveness and smiling are symptoms indicative of a right hemisphere stroke. Aphasia, cautiousness, the inability to discriminate words, quick to anger, and frustration are symptoms indicative of a left hemisphere stroke.

An older adult client is hospitalized with Guillain-Barr syndrome. The client is given amitriptyline (Elavil). After receiving the hand-off report, what actions by the nurse are most important? (Select all that apply.) a. Administering the medication as ordered b. Advising the client to have help getting up c. Consulting the provider about the drug d. Cutting the dose of the drug in half e. Placing the client on safety precautions

ANS: B, C, E Amitriptyline is a tricyclic antidepressant and is considered inappropriate for use in older clients due to concerns of anticholinergic effects, confusion, and safety risks. The nurse should tell the client to have help getting up, place the client on safety precautions, and consult the provider. Since this drug is not appropriate for older clients, cutting the dose in half is not warranted.

An older adult client is hospitalized with Guillain-Barré syndrome. The client is given amitriptyline (Elavil). After receiving the hand-off report, what actions by the nurse are most important? (Select all that apply.) a. Administering the medication as ordered b. Advising the client to have help getting up c. Consulting the provider about the drug d. Cutting the dose of the drug in half e. Placing the client on safety precautions

ANS: B, C, E Amitriptyline is a tricyclic antidepressant and is considered inappropriate for use in older clients due to concerns of anticholinergic effects, confusion, and safety risks. The nurse should tell the client to have help getting up, place the client on safety precautions, and consult the provider. Since this drug is not appropriate for older clients, cutting the dose in half is not warranted.

A client is admitted with Guillain-Barré syndrome (GBS). What assessment takes priority? a. Bladder control b. Cognitive perception c. Respiratory system d. Sensory functions

ANS: C Clients with GBS have muscle weakness, possibly to the point of paralysis. If respiratory muscles are paralyzed, the client may need mechanical ventilation, so the respiratory system is the priority. The nurse will complete urinary, cognitive, and sensory assessments as part of a thorough evaluation.

A client who has Guillain-Barré syndrome is scheduled for plasmapheresis. Before the procedure, which clinical manifestation does the nurse use to determine patency of the client's arteriovenous shunt?a.Palpable distal pulses b.A pink, warm extremity c.The presence of a bruit d.Shunt pressure higher than 25 mm Hg

ANS: C Nursing care of the client undergoing plasmapheresis includes care of the shunt. The nurse checks for bruits every 2 to 4 hours for patency. Pulse and extremity assessments do not provide information related to shunt patency. Pressure within the shunt is not tested before treatment to determine patency.

A nurse is caring for four clients in the neurologic/neurosurgical intensive care unit. Which client should the nurse assess first? a. Client who has been diagnosed with meningitis with a fever of 101 F (38.3 C) b. Client who had a transient ischemic attack and is waiting for teaching on clopidogrel (Plavix) c. Client receiving tissue plasminogen activator (t-PA) who has a change in respiratory pattern and rate d. Client who is waiting for subarachnoid bolt insertion with the consent form already signed

ANS: C The client receiving t-PA has a change in neurologic status while receiving this fibrinolytic therapy. The nurse assesses this client first as he or she may have an intracerebral bleed. The client with meningitis has expected manifestations. The client waiting for discharge teaching is a lower priority. The client waiting for surgery can be assessed quickly after the nurse sees the client who is receiving t-PA, or the nurse could delegate checking on this client to another nurse.

The nurse assesses a client who has Guillain-Barré syndrome. Which clinical manifestation does the nurse expect to find in this client? a.Ophthalmoplegia and diplopia b.Progressive weakness without sensory involvement c.Progressive, ascending weakness and paresthesia d.Weakness of the face, jaw, and sternocleidomastoid muscles

ANS: C The most common clinical pattern of Guillain-Barré syndrome is the ascending variety. Weakness and paresthesia begin in the lower extremities and progress upward. The other manifestations are not associated with Guillain-Barré syndrome.

A client has a traumatic brain injury and a positive halo sign. The client is in the intensive care unit, sedated and on a ventilator, and is in critical but stable condition. What collaborative problem takes priority at this time? a. Inability to communicate b. Nutritional deficit c. Risk for acquiring an infection d. Risk for skin breakdown

ANS: C The positive halo sign indicates a leak of cerebrospinal fluid. This places the client at high risk of acquiring an infection. Communication and nutrition are not priorities compared with preventing a brain infection. The client has a definite risk for a skin breakdown, but it is not the immediate danger a brain infection would be.

The nurse is teaching the spouse and client who has had a brain attack about rehabilitation. Which statement by the spouse demonstrates understanding of the nurse's instruction? a. "Rehabilitation and physical therapy are the same thing." b. "Frequent stimulation will help with the rehabilitation process." c. "The rehabilitation therapist will help identify changes needed at home." d. "My spouse will no longer need to take blood pressure medication."

ANS: C The rehabilitation therapist and home health professionals assist the client and family in adapting the home environment to the client's needs and assess the client's need for therapy. Any medication regimen for the client must be maintained. Rehabilitation is much more comprehensive than physical therapy. The family should develop a home routine that provides structure, repetition, and consistency.

The nurse is teaching a client and family about home care after a stroke. Which statement made by the client's spouse indicates a need for further teaching? a. "My husband must take his medicine every day to prevent another stroke." b. "The physical therapist will show us how to use the equipment so my husband can climb the stairs and get into and out of bed." c. "I should spend all my time with my husband in case I'm needed." d. "My husband may get depressed."

ANS: C Family members can start to feel socially isolated when caring for a loved one. The family may need to plan for regular respite care in a structured day-care respite program or through relief provided by a friend or neighbor. The life changes associated with stroke often cause a change in the client's self-esteem. The client who has had a stroke should maintain a regular medication regimen, such as anticoagulant therapy, to prevent another stroke. Once the home health nurse has assessed the home environment, he or she will notify the health care provider of the need for ancillary services, such as a physical therapist. The physical therapist will identify adaptive equipment needs, will request them, and then will instruct the client about their use, along with developing an exercise program.

A client is being discharged home after surgery for brain cancer. Which statement by the client's spouse indicates a correct understanding of the nurse's discharge teaching? a. "Life will be back to normal soon." b. "I will have to quit my job to care for my spouse." c. "The case manager will provide home care." d. "We can find a support group through the local American Cancer Society."

ANS: D The American Cancer Society is a good community resource for clients with malignant tumors and their families. It is not a requirement that the client's spouse quit his or her job; resources are available for in-home client care. Although life need not be completely altered, it will never revert back to normal, as it was before the client's cancer occurred. The case manager helps coordinate care and teaching, but does not provide home care.

An alert and oriented person is admitted to the emergency department with a GCS of 10, indicating a moderate brain injury. Which assessment finding will the nurse report immediately to the health care provider? A. Photophobia accompanied by headache B. New onset of dizziness when lying quietly in bed C. A brisk pupillary reaction to light D. New difficulty in responsiveness or sudden drowsiness

ANS: D DRationale: The change in level of consciousness is the most sensitive indicator of new or worsening brain damage and must be communicated urgently to the health care provider. Any deterioration in alertness or responsiveness in a client with new brain injury is an emergency. Although photophobia and dizziness are concerning, they are not emergencies. A brisk pupillary response is normal.

A client in the emergency department is having a stroke. The client weighs 225 pounds. After the initial bolus of t-Pa, at what rate should the nurse set the IV pump? (Record your answer using a decimal rounded to the nearest tenth.) ____ mL/hr

ANS:1.4 mL/hrThe client weighs 102 kg. The dose of t-PA is 0.9 mg/kg with a maximum of 90 mg, so the clients dose is 90 mg.10% of the dose is given as a bolus IV over the first minute (9 mg). That leaves 81 mg to run in over 59 minutes., which rounds to 1.4 mL/hr.

A client in the emergency department is having a stroke and the provider has prescribed the tissue plasminogen activator (t-PA) alteplase (Activase). The client weighs 146 pounds. How much medication will this client receive? (Record your answer using a whole number.) _____ mg

ANS:60 mg The dose of t-PA is 0.9 mg/kg with a maximum dose of 90 mg. The client weighs 66.4 kg.0.9 mg 66.4 = 59.76 mg, which rounds to 60 mg.

A 20-year-old client who fell approximately 30' is unresponsive and breathless. A cervical spine injury is suspected. How should the first-responder open the client's airway for rescue breathing?

By performing a jaw-thrust maneuver If the client has a suspected cervical spine injury, a jaw-thrust maneuver should be used to open the airway. If the tongue or relaxed throat muscles are obstructing the airway, a nasopharyngeal or oropharyngeal airway can be inserted; however, the client must have spontaneous respirations when the airway is open. The head-tilt, chin-lift maneuver requires neck hyperextension, which can worsen the cervical spine injury.

A client receiving vent-assisted mode ventilation begins to experience cluster breathing after recent intracranial occipital bleeding. Which action would be most appropriate?

Call the physician while another nurse checks the vital signs and ascertains the patient's Glasgow Coma score. Cluster breathing consists of clusters of irregular breaths followed by periods of apnea on an irregular basis. A lesion in the upper medulla or lower pons is usually the cause of cluster breathing. Because the client had a bleed in the occipital lobe, which is superior and posterior to the pons and medulla, clinical manifestations that indicate a new lesion are monitored very closely in case another bleed ensues. The physician is notified immediately so that treatment can begin before respirations cease. Another nurse needs to assess vital signs and score the client according to the GCS, but time is also of the essence. Checking deep tendon reflexes is one part of the GCS analysis.

The nurse is preparing the client diagnosed with a head injury for a magnetic resonance imaging (MRI). Which interventions should the nurse implement? Select all that apply. l 1. Ask the client if he/she is claustrophobic. 2. Have the client sign a procedural permit. 3. Determine if the client is allergic to shellfish. 4. Check if the client has any prosthetic devices. 5. Ask the client to empty his/her bladder

Correct answer 1, 4, 5: The client is enclosed in an MRI tube for an extended period so the client cannot be claustrophobic or want to stop the procedure. An MRI cannot be completed on a client with a metal prosthesis unless it is made with titanium because the MRI may dislodge the prosthesis. The hospital admission permit covers the MRI, and because no contrast dye is now used in most MRIs, an allergy to shellfish is not pertinent

The nurse is preparing for a lumbar puncture for the client diagnosed with R/O meningitis. Which interventions should the nurse implement? Select all that apply. 1. Determine if the client has any allergies to iodine. 2 .Do not let the client urinate 2 hours before the procedure. 3. Place the client in a prone position with the face turned to the side. 4. Instruct the client to take slow deep breaths during the procedure. 5. Label the specimen and send to the laboratory for cultures.

Correct answer 1, 4, 5: The lumbar area is cleansed with Betadine; therefore, iodine allergies should be noted. The client's bladder should be empty for comfort during the procedure, and the client should be in a side-lying position with back arched for access to intravertebral space. Taking slow deep breaths will help calm the client, and specimens are sent to the laboratory.

Which type of precautions should the nurse implement for the client diagnosed with aseptic meningitis? l 1. Standard precautions .l 2. Airborne precautions. l 3. Contact precautions. l 4. Droplet precautions.

Correct answer 1: Aseptic meningitis is caused by a noninfectious agent or a virus and is not likely to be transmitted to other people; therefore, standard precautions would be expected. Septic meningitis would require droplet precautions for 24-48 hours after initiation of antibiotics.

The nurse is planning care for the client experiencing dysphagia secondary to a CVA. Which intervention should be included in the plan of care? l 1. Evaluate the client during mealtime. l 2. Position the client in a semi-Fowler position. l 3. Administer oxygen during meals. l 4. Refer the client to a physical therapist.

Correct answer 1: Dysphagia (swallowing difficulty) puts the client at risk for aspiration, pneumonia, dehydration, and malnutrition; therefore, the nurse should evaluate the client during mealtime. The client should be in a high Fowler position or, preferably, in a chair.

The intensive care nurse is caring for a client diagnosed with a TBI who is exhibiting decorticate posturing. Three hours later the client has flaccid posturing. Which action should the nurse implement first? l1. Notify the client's health-care provider (HCP) immediately. l2. Prepare to administer mannitol (Osmitrol), an osmotic diuretic. l3. Complete a thorough neurological assessment on the client. l 4. Reassess the client in 1 hour, including calculating the Glasgow Coma Scale.

Correct answer 1: Flaccid posturing is the worst-case scenario for a client with a TBI; therefore, the nurse should notify the HCP. Completing a neurological assessment, administering an osmotic diuretic, and reassessing the client are all plausible interventions, but they are not the first to be implemented

The client diagnosed with a brain tumor was admitted to the ICU with decorticate posturing. Which indicates that the client's condition is improving? 1. The client has purposeful movement with painful stimuli. 2 .The client assumes adduction of the upper extremities. 3. The client assumes the decerebrate posture upon painful stimuli. 4. The client has become flaccid and does not respond to stimuli.

Correct answer 1: Purposeful movement following painful stimuli would indicate an improvement in the client's condition. Adducting the upper extremities while internally rotating the lower extremities is decorticate positioning; this would indicate the client's condition had not changed. Decerebrate posturing and flaccid movement indicate a worsening of the condition

The nurse is admitting a client diagnosed with meningococcal meningitis and notes lesions over the face and extremities. Which priority intervention should the nurse implement? l1. Initiate the intravenous antibiotics stat. l2. Obtain a skin biopsy for culture and sensitivity. l3. Perform a complete neurological assessment. l4. Close all the curtains in the room and turn off lights.

Correct answer 1: Purpuric lesions over the face and extremities are the signs of a fulminating infection in clients with meningococcal meningitis. The infection can lead to death within a few hours. The nurse should start the antibiotics immediately.

The client has sustained a traumatic brain injury (TBI) secondary to a motor vehicle accident. Which signs/symptoms would the emergency department (ED) nurse expect the client to exhibit? l 1. Blurred vision, nausea, and right-sided hemiparesis. l 2. Increased urinary output, negative Babinski, and ptosis.l 3. Autonomic dysreflexia, positive Brudzinski, and hyperpyrexia.l 4. Negative dextrostik, nuchal rigidity, and nystagmus.

Correct answer 1: Signs/symptoms of TBI include neurological deficits, among them blurred vision, nausea, and right-sided hemiparesis. A positive Babinski sign would also occur with head trauma. Autonomic dysreflexia would be found in a client with a spinal cord injury; a positive dextrostik for glucose would be found in someone with a cerebrospinal fluid leak; and a positive Brudzinski and nuchal rigidity are signs of meningitis

Which clinical manifestation would the nurse assess in the client with a T-12 spinal cord injury (SCI) who is experiencing spinal shock?l 1. Flaccid paralysis below the waist. l 2. Lower extremity muscle spasticity. l 3. Complaints of a pounding headache. l 4. Hypertension and bradycardia.

Correct answer 1: Spinal shock is associated with an SCI. It is a sudden depression of reflex activity, a loss of sensation, and flaccid paralysis below the level of the injury. T-12 is just above the waist.

The nurse is discussing the TBI Act at a support group meeting. Which statement best explains the act? 1. It is a federal act that provides public policy regarding community living for clients with a TBI. 2. It ensures that all public buildings must have access for physically challenged clients. 3. This act ensures that all clients with a TBI have access to rehabilitation services. 4. It is a national policy that establishes guidelines for neurological rehabilitation centers.

Correct answer 1: The TBI Act is part of the Children's Act of 2000 and is the only federal legislation designed for clients with a TBI. The Act provides for a balanced public policy for prevention, education, research, and community living for clients with a TBI and their families.

The male client with a C-6 SCI tells the home health nurse he has had a severe pounding headache for the last 2 hours. Which intervention should the clinic nurse implement? l 1. Determine when and how much the client last urinated. l 2. Ask the client if he has taken any medication for the headache. l 3. Inquire when the client had his last bowel movement. l 4. Check the client's respiratory rate reading immediately.

Correct answer 1: The cause of the pounding headache is most likely autonomic dysreflexia, a result of exaggerated autonomic responses to stimuli. An elevated blood pressure would confirm this.The most common cause of autonomic dysreflexiais a full bladder. All the other options could be implemented, but confirming the autonomic dysreflexia is priority.

The nurse is caring for clients on a medical surgical floor. Which client should be assessed first? l 1. The client diagnosed with epilepsy who reports over the intercom having an aura. l 2. The client with an L-1 SCI who is complaining of shortness of breath while exercising. l 3. The client diagnosed with Parkinson disease who is being discharged today .l 4. The client diagnosed with a CVA who has resolving left hemiparesis.

Correct answer 1: The client with an aura is getting ready to have a seizure. This client should be seen first.

The nurse on the rehabilitation unit is caring for the following clients with SCIs. Which client should the nurse assess first after receiving the change-of-shift report? l1. The client with a C-6 SCI who has a warm, reddened edematous gastrocnemius muscle. l 2. The client with an L-4 SCI who is concerned about being able to live independently. l 3. The client with an L-2 SCI who is complaining of a headache and nausea. l 4. The client with a T-4 SCI who is unable to move the lower extremities.

Correct answer 1: The gastrocnemius muscle is the calf muscle, and warmth, redness, and swelling in the muscles indicate the client has a deep vein thrombosis (DVT), which requires immediate intervention. A client with an L-2 SCI (option 3) would not experience autonomic dysreflexia. A client with a T-4 SCI (option 4) would not be expected to be able to move the lower extremities.

The HCP has discussed a carotid endarterectomy with the client who has experienced two transient ischemic attacks (TIAs). The client tells the nurse, "I really don't understand why I need this procedure, and I don't want to have it." Which scientific rationale would support the nurse's response? 1. This surgery is indicated for clients with symptoms of a TIA due to carotid artery stenosis. 2. This surgical procedure will ensure the client does not have a cerebrovascular accident. 3. This surgery will remove all atherosclerotic plaque from the carotid arteries. 4. This surgical procedure will increase the elasticity of the carotid arterial wall.

Correct answer 1: This is the rationale the nurse would utilize to encourage the client to have this surgical procedure. An endartectomy does not ensure the client will not have a CVA nor does it ensure that all atherosclerotic plaque will be removed or that the carotid artery wall will become more elastic.

The intensive care nurse is caring for a client following an infratentorial craniotomy. Which interventions should the nurse implement? Select all that apply .l 1. Keep the head of the bed elevated at 30 degrees. l 2. Keep a humidifier in the client's room. l 3. Do not put anything in the client's mouth. l 4. Provide the client with a clear liquid diet .l 5. Assess the client's respiratory status every hour.

Correct answer 2, 4, 5: Humidified air would be provided; the client's diet is started slowly; and the respiratory status is assessed because the centers that control respiration and vomiting are in the area of the brain affected by the surgery. The head of the bed would be flat, and caution with oral care is appropriate for a client with a transsphenoidal hypophysectomy, not with an infratentorial craniotomy.

The intensive care nurse is caring for a client diagnosed with a closed head injury. Which data would warrant immediate intervention?l 1. The client refuses to cough and deep-breathe.l 2. The client's Glasgow Coma Scale goes from 13 to 7.l 3. The client complains of a frontal headache. l 4. The client's Mini-Mental Status Exam (MMSE) is 30.

Correct answer 2: A 15 on the Glasgow Coma Scale indicates the client is neurologically intact; a decrease to 7 indicates an increase in the intracranial pressure, which warrants immediate intervention. A 30 on the MMSE indicates the client is cognitively intact.

The nurse is assessing the client experiencing a left-sided cerebrovascular accident (CVA). Which clinical manifestations would the nurse expect the client to exhibit? l 1. Hemiparesis of the left arm and apraxia. l 2. Paralysis of the right side of the body and aphasia. l 3. Inability to recognize and use familiar objects. l 4. Impulsive behavior and hostility toward family.

Correct answer 2: A left-sided CVA results in right- sided paralysis, right visual field deficit, aphasia (inability to speak), and altered intellectual ability. All other options are results of right-sided CVA.

The charge nurse has received laboratory data for clients. Which situation requires the charge nurse's intervention first? l1. The client with a brain tumor who has ABGs: ph 7.36, PaO2 95, PaCO2 38, HCO3 24. l2. The postoperative craniotomy client who has a serum sodium level of 153 mEq/L. l3. The client with septic meningitis who has a white blood cell count of 12,000 mm. l4. The client with epilepsy who has a serum phenytoin (Dilantin) level 15 mcg/mL.

Correct answer 2: An elevated serum sodium level (normal is 135-145 mEq/L) indicates possible diabetes insipidus, which is a complication of brain surgery. The ABGs are within normal limits, the WBC count would be elevated in a client with meningitis, and the therapeutic Dilantin level is 10-20 mcg/mL.

The client diagnosed with septic meningitis is admitted to the medical floor at 1200. Which HCP's order would the nurse implement first? l 1. Administer intravenous antibiotic. l 2. Start the client's intravenous line. l 3. Provide a quiet, calm dark room. l 4. Initiate seizure precautions.

Correct answer 2: Intravenous antibiotics are of paramount importance, so the nurse must start an intravenous line first.

The client diagnosed with a right-sided CVA is admitted to the rehabilitation unit. Which intervention should be included in the nursing care plan? l 1. Turn and reposition the client every shift. l 2. Place a small pillow under the client's left shoulder. l 3. Have the client perform quadriceps exercises three times a day .l 4. Instruct the client to hold fingers in a fist.

Correct answer 2: Placing a small pillow under the left shoulder will prevent the shoulder from adducting toward the chest and developing a contracture. The client should be repositioned at least every 2 hours; quadricep exercises should be done for 10 minutes at least five times a day; and the fingers are positioned so that they are barely flexed.

The client is being admitted with rule-out (R/O) brain tumor. Which signs/symptoms support the diagnosis of a brain tumor?l 1. Widening pulse pressure, hypertension, and bradycardia. l 2. Headache, vomiting, and diplopia l 3. Hypotension, tachycardia, and tachypnea. l 4. Abrupt loss of motor function, diarrhea, and changes in taste.

Correct answer 2: The classic triad of symptoms ofa brain tumor includes a headache that is dull and unrelenting and worse in the morning, vomiting unrelated to food intake, and edema of the optic nerve (papilledema) causing diplopia. Option 1 is the Cushing triad, which indicates increased intracranial pressure that would not be seen initially on diagnosis; option 3 is signs/symptoms of hypovolemic shock.

The rehabilitation nurse caring for the young client with a T-12 SCI is developing the nursing care plan. Which priority intervention should the nurse implement? 1. Monitor the client's indwelling urinary catheter. 2. Insert a rectal stimulant at the same time every morning. 3. Encourage active lower extremity range of motion (ROM) exercises. 4. Refer the client to a vocational training assistance program.

Correct answer 2: The client's bowel and bladder functions must be addressed; therefore, administering a daily rectal stimulant will ensure a daily bowel movement. Indwelling urinary catheters are discouraged due to the increased risk of infection associated with their use.

The client has undergone a craniotomy for a brain tumor. Which data indicate a complication of this surgery? 1. The client complains of a headache at a 3-4 on a 1-10 scale. 2. The client has a urinary output of 250 mL over the last 24 hours. 3. The client has a serum sodium level of 137 mEq/L. 4. The client experiences dizziness when trying to get up too quickly.

Correct answer 2: The decreased urinary output may indicate syndrome of inappropriate antidiuretic hormone (SIADH), which is a complication of a craniotomy. A headache after this surgery would be an expected occurrence. The sodium level is normal (135-145 mEq/L). Dizziness upon arising quickly would not be a complication of this surgery

The nurse has received the morning shift report. Which client should the nurse assess first? 1. The client who is complaining of a headache at a 3 on a scale of 1-10. 2. The client who has an apical pulse of 56 and a blood pressure of 210/116. 3. The client who is reporting not having a bowel movement in 3 days. 4. The client who is angry because the call light was not answered for 1 hour.

Correct answer 2: This blood pressure is extremely high, and the pulse rate is decreased; therefore, this client should be assessed first. A 3 headache, no bowel movement, and an upset client would not be priority over a client who may be having a CVA

The nurse is caring for a client diagnosed with septic meningitis. The UAP reports T 101.6°F, P 128, R 32, B/P 96/46. Which action should the nurse implement first? l1. Notify the HCP. l2. Assess the client immediately. l3. Prepare to administer acetaminophen (Tylenol). l4. Check the chart for the culture and sensitivity report.

Correct answer 2: Whenever another health-care team member reports information to the nurse, assessment should be completed to confirm the data. Then the nurse should notify the HCP, administer Tylenol to decrease the fever, and check the chart, but the nurse must first realize this is potential septic shock, and the client should be assessed.

The nurse is preparing to administer dexamethasone (Decadron) intravenous push (IVP) to a client with an acute spinal cord injury. Which interventions should the nurse implement? Rank in order. l1. Administer the medication over 2 minutes. l2. Dilute the medication with normal saline. l3. Check the client's medication administration record (MAR). l4. Check the client's identification band. l5. Clamp the primary tubing distal to the port.

Correct answer 3, 2, 4, 5, 1: First check the MAR to ensure the right medication, the right dose, at the right time. Diluting the medication saves the vein and decreases the client's pain during administration. Check for the right client by checking the client's identification band. Clamping the tubing will ensure the medication goes into the vein, and 2 minutes is the recommended administration time

The nurse is caring for a female client who sustained a closed head injury 8 days ago due to a motor vehicle accident. Which signs/symptoms would alert the nurse to a complication of the head injury? 1. The client reports having trouble sleeping due to having nightmares about the wreck. 2. The client tells the nurse she has a stuffy nose and green nasal drainage. 3. The client complains of extreme thirst and has an increased urine output. 4. The client informs the nurse that she has started her menstrual period.

Correct answer 3: For 7-10 days post head injury, the client is at risk for developing diabetes insipidus, which is a lack of the antidiuretic hormone, resulting in increased urine output and increased thirst.

The nurse is assessing the client diagnosed with bacterial meningitis. In addition to nuchal rigidity, which clinical manifestations would the nurse assess? l 1. Positive Cushing sign and ascending paralysis. l 2. Negative Kernig sign and facial tingling. l 3. Positive Brudzinski sign and photophobia. l 4. Negative Trousseau sign and descending paralysis.

Correct answer 3: A positive Brudzinski sign (raise the client's head, and the knees will come up) and photophobia due to meningeal irritation are key signs of meningitis. A positive Kernig sign (client is unable to extend leg when lying flat) would also be expected.

The client diagnosed with chronic atrial fibrillation has experienced a transient TIA. Which discharge instruction should the nurse implement? l1. Keep nitroglycerin tablets in a dark-colored bottle. l2. Check the radial pulse prior to all medications. l3. Obtain International Normalized Ratio (INR) routinely. l4. Take over-the-counter vitamin K tablets daily.

Correct answer 3: An oral anticoagulant, warfarin (Coumadin), will be prescribed to help prevent the formation of thrombi in the atrium secondary to atrial fibrillation. The thrombi can become embolic, which may cause a TIA. The INR is the laboratory value used to determine therapeutic oral anticoagulant levels

The client is diagnosed with a pituitary tumor and is scheduled for a transsphenoidal hypophysectomy. Which postoperative instruction is important to discuss with the client? l 1. Demonstrate to a family member how to change a turban dressing. l 2. Explain to the client how to monitor urine output at home. l 3. Tell the client not to blow his nose for 2 weeks after surgery. l 4. Tell the client he will have to lie flat for 24 hours following the surgery.

Correct answer 3: Blowing the nose creates increased intracranial pressure and could result in a leak of cerebral spinal fluid. A transsphenoidal hypophysectomy is done by an incision above the gum line, and there is no turban dressing. The head of the bed is elevated to 30 degrees to allow for gravity to assist in draining the cerebrospinal fluid.

Which client would the nurse identify as being least at risk for experiencing a CVA? l 1. A 55-year-old African-American male who is obese. l 2. A 73-year-old Japanese female who has essential hypertension. l 3. A 67-year-old Caucasian male whose cholesterol level is below 200 mg/dL. l 4. A 39-year-old female who is taking oral contraceptives.

Correct answer 3: Caucasians have a lower risk of CVA than African Americans, Hispanics, and Native Pacific Islanders. A high cholesterol level, being African American, hypertension, and oral contraceptive use are risk factors for developing a CVA

The nurse is caring for a client with a C-6 SCI in the neurological intensive care unit. Which nursing intervention should be implemented? l 1. Monitor the client's heparin drip. l 2. Assess the neurological status every shift. l 3. Maintain the client's ice saline infusion. l 4. Administer corticosteroids intrathecally.

Correct answer 3: Current treatment options that have proven efficacy in treating SCI is to decrease inflammation and edema by lowering the body temperature with ice saline solutions. Intravenous corticosteroid therapy is a standard of care but not intrathecal, into the spinal cord

The nurse asks the UAP to help admit the client diagnosed with bacterial meningitis. Which nursing task is priority? l 1. Take the client's vital signs. l 2. Obtain the client's height and weight. l 3. Prepare the room for respiratory isolation. l 4. Pull the drapes and make sure the room is dim

Correct answer 3: Equipment needed for the staff to enter the client's room safely is the priority nursing task that can be delegated. All other tasks could be safely delegated to the UAP, but they are not priority.

The male client diagnosed with a brain tumor is having a closed magnetic resonance imaging (MRI) scan in 1 hour. The client tells the radiology nurse, "I don't like small enclosed spaces." Which action should the nurse implement? l 1. Allow the client to express his feelings. l 2. Discuss the procedure with the client. l 3. Obtain an order for an anti-anxiety medication. l 4. Reschedule the procedure for another day.

Correct answer 3: The client is claustrophobic and will need medications to help decrease the anxiety associated with small enclosed spaces. Ventilating feelings and discussing the procedure will not help claustrophobia. Reschedule for an open MRI, not another closed MRI.

The nurse is developing a plan of care for a client diagnosed with septic meningitis. Which client goal would be most appropriate for the client problem of "altered thermoregulation"? 1. The client will have no injury from using the hypothermia blanket. 2. The client will be protected from injury if seizure activity occurs. 3. The client will be afebrile for 48 hours prior to discharge. 4. The client will have serum electrolytes within normal limits.

Correct answer 3: The client with septic meningitis has a high fever; therefore, being afebrile for 48 hours would be an appropriate goal.

The 88-year-old client is admitted to the ED with numbness and weakness of the left arm and slurred speech. The computed tomography (CT) scan was negative for bleeding. Which nursing intervention is priority? l1. Prepare to administer tissue plasminogen activator (TPA). l2. Discuss the precipitating factors that caused the symptoms. l3. Determine the exact time the symptoms occurred. l4. Notify the speech pathologist for an emergency consult.

Correct answer 3: The nurse must first determine when the symptoms started before administering TPA, a standard of care. TPA must be initiated within 3 hours of the start of symptoms because, after that time, revascularization of necrotic tissue, which occurs with the administration of TPA, increases the risk for cerebral edema and hemorrhage.

The emergency department nurse is entering the room of a client who was at a baseball game and was hit in the head with a bat. Which intervention should the nurse implement first? l 1. Assess the client's orientation to date, time, and place. l 2. Ask the client to squeeze the nurse's fingers.l 3. Determine the client's reaction to the door opening. l 4. Request the client to move his lower legs.

Correct answer 3: The nurse should first determine how alert the client is by noticing the reaction when the door opens. The best reaction is spontaneous opening of the eyes without verbal or noxious stimuli. The other three options are appropriate but should not be the nurse's first intervention when entering the client's room.

The nurse is caring for a client who has a C-6 vertebral fracture and is using Crutchfield tongs with 2-pound weights. Which data would the nurse expect the client to exhibit? l 1. The client is on controlled mechanical ventilation at 12 respirations a minute. l 2. The client has no movement of the lower extremities. l 3. The client has 2+ deep tendon reflexes in the lower extremities. l 4. The client has loss of sensation below the C-6 vertebral fracture.

Correct answer 3: The spinal cord has not been injured; therefore, normal body movement, responses, and reflexes should be intact. The Crutchfield tongs ensure that the cervical spine remains in alignmen

The male client is scheduled for gamma knife stereotactic surgery for a brain tumor. Which preoperative instruction should the nurse discuss with the client? l1. Instruct the client to avoid bright lights and wear sunscreen. l2. Tell the client he must sleep with the head of the bed elevated. l3. Explain there are no activity limitations after this procedure. l 4. Encourage the client to take off at least 2 weeks from work.

Correct answer 3: This is a day-surgery procedure, and the client is usually discharged home 3-4 hours after the surgery and can resume normal activities.

The rehabilitation nurse is caring for the client with a closed head injury. Which cognitive goal would be most appropriate for this client? 1. The client will be able to feed himself/herself independently. 2. The client will attend therapy sessions 3 hours a day. 3. The client will interact appropriately with staff members. 4. The client will be able to stay on task for 15 minutes.

Correct answer 4: Cognitive is mental functioning; therefore, the ability to stay on task would be the client's most appropriate cognitive goal

The client diagnosed with a brain tumor is prescribed intravenous dexamethasone (Decadron), a steroid.Which intervention should the nurse implement when administering this medication? l 1. Administer medication with normal saline only. l 2. Check the client's white blood cell (WBC) count. l 3. Determine if the client has oral candidiasis. l 4. Monitor the client's glucose level.

Correct answer 4: Decadron, a glucocorticosteroid, will increase insulin resistance, which increases glucose levels; therefore, glucose levels should be monitored. Decadron is compatible with dextrose, so normal saline does not need to be used, and the WBC count and oral candidiasis would not be interventions pertinent to administering this medication.

The client diagnosed with a CVA has hemiparesis. Which problem would be priority for the client? l 1. Impaired skin integrity. l 2. Fluid volume overload. l 3. High risk for aspiration. l 4. High risk for injury.

Correct answer 4: Hemiparesis is a weakness on one side of the body that may lead to falls; this makes high risk for injury the priority problem for this client.

The 18-year-old client is admitted to the medical floor with a diagnosis of meningitis. Which priority intervention should the nurse assess? l 1. Assess the client's neurovascular status. l 2. Assess the client's cranial nerve IX function. l 3. Assess the client's brachioradialis reflex. l 4. Assess the client's neurological status.

Correct answer 4: Meningitis directly affectsthe client's brain; therefore, assessing the neurological status would have priority for this client. Neurovascular assessment involves peripheral nerves and changes such as paralysis and skin temperature.

A college student came to the university health clinic and was diagnosed with bacterial meningitis and admitted to a local hospital. Which intervention should the university health clinic nurse implement? l 1. Place the client's dormitory under strict respiratory isolation. l2. Notify the parents of all students about the meningitis outbreak. l3. Arrange for students to receive the meningococcal vaccination. l4. Ensure dormitory roommates receive chemoprophylaxis using rifampin.

Correct answer 4: People in close contact with clients diagnosed with meningococcal meningitis, the most common type of infectious agent in group settings, should receive chemoprophylaxis for prevention of meningitis. The public health nurse or college administration would notify parents. It is too late for the vaccine

The client is diagnosed with a frontal lobe brain tumor. Which sign/symptom would the nurse expect the client to exhibit? l 1. Ataxia. l 2. Decreased visual acuity. l 3. Scanning speech. l 4. Personality changes.

Correct answer 4: Personality changes occur in a client with a frontal lobe tumor. Ataxia or gait problems indicate a temporal lobe tumor. Decreased visual acuity is a symptom indicating papilledema, a general symptom of the majority of all brain tumors, not specifically a frontal lobe tumor. Scanning speech is symptomatic of multiple sclerosis

The client with a T-1 SCI complains of lightheadedness and dizziness when the head of the bed is elevated. The client's B/P is 84/40. Which action should the nurse implement first? l 1. Increase the client's intravenous (IV) rate by 50 mL/hr. l 2. Administer dopamine, a vasopressor, via an IV pump. l 3. Notify the HCP immediately. l 4. Lower the client's head of bed immediately

Correct answer 4: The blood pressure tends to be very unstable and low for clients with an SCI of T-6 or above, and slight elevations of the head of the bed can cause profound drops in the client's vital signs.

The nurse is admitting a client diagnosed with meningitis who has AIDS. Which signs/symptoms would the nurse expect the client to exhibit? l 1. A positive Babinski sign. l 2. Diplopia and blurred vision. l 3. Auditory deficits. l 4. The client may be asymptomatic.

Correct answer 4: The client with AIDS may be asymptomatic or may exhibit atypical symptoms because of blunted inflammatory responses.

The client with increased intracranial pressure is receiving mannitol (Osmitrol), an osmotic diuretic. Which intervention should the nurse implement? l1. Monitor the client's complete blood cell (CBC) count. l2. Do not administer the drug if the client's apical pulse is less than 60. l3. Ensure that the client's cardiac status is monitored by telemetry. l4. Use a filter needle when administering the medication.

Correct answer 4: The nurse must use a filter needle when administering mannitol because crystals may form in the solution and syringe and be inadvertently injected into the client. The CBC and apical pulse are not affected by the medication. Mannitol is administered cautiously in clients with heart failure, but telemetry is not required routinely.

The male client is being discharged from the ED after sustaining a minor head injury. Which statement indicates the wife understands the discharge teaching? l1. "My husband will be hard to wake up for a couple of days." l2. "He doesn't need any pain medication because I have some at home." l3. "I should not give my husband anything to eat or drink for 12 hours." l4. "I will bring my husband back to the emergency room if he starts vomiting."

Correct answer 4: Vomiting indicates an increase in intracranial pressure, which is a complication of a head injury. The client should arouse easily, may eat and drink (not alcohol), and should not take any type of pain medication that would mask mental status.

A client will be receiving plasmapheresis for treatment of Guillain-Barre'syndrome (GBS). Which posttreatment test will the nurse anticipate to be ordered?

Electrolyte panel For the client receiving plasmapheresis for treatment of GBS, the nurse expects that an electrolyte panel will be ordered. Electrolytes will be checked since citrate-induced hypocalcemia is a complication of plasmapheresis.An electroencephalogram evaluates brain waves and is useful in detecting seizure activity. It would not be beneficial in this situation. A lumbar puncture might have been performed as part of the diagnostic process initially but not as part of posttreatment. There is no role for a urinalysis after plasmapheresis.

The nurse is caring for a client with a T5 complete spinal cord injury. Upon assessment, the nurse notes flushed skin, diaphoresis above the T5, and a blood pressure of 162/96. The client reports a severe, pounding headache. Which of the following nursing interventions would be appropriate for this client? Select all that apply.

Elevate the HOB to 90 degrees Loosen constrictive clothing Assess for bladder distention and bowel impaction Administer antihypertensive medication --The client has signs and symptoms of autonomic dysreflexia. The potentially life-threatening condition is caused by an uninhibited response from the sympathetic nervous system resulting from a lack of control over the autonomic nervous system. The nurse should immediately elevate the HOB to 90 degrees and place extremities dependently to decrease venous return to the heart and increase venous return from the brain. Because tactile stimuli can trigger autonomic dysreflexia, any constrictive clothing should be loosened. The nurse should also assess for distended bladder and bowel impaction, which may trigger autonomic dysreflexia, and correct any problems. Elevated blood pressure is the most life-threatening complication of autonomic dysreflexia because it can cause stroke, MI, or seizures. If removing the triggering event doesn't reduce the client's blood pressure, IV antihypertensives should be administered. A fan shouldn't be used because cold drafts may trigger autonomic dysreflexia.

In planning the care for a client who has had a posterior fossa (infratentorial) craniotomy, which of the following is contraindicated when positioning the client?

Elevating the head of the bed to 30 degrees Elevating the HOB to 30 degrees is contraindicated for infratentorial craniotomies because it could cause herniation of the brain down onto the brainstem and spinal cord, resulting in sudden death. Elevation of the head of the bed to 30 degrees with the head turned to the side opposite of the incision, if not contraindicated by the ICP; is used for supratentorial craniotomies.

Which of the following nursing interventions is appropriate for a client with an ICP of 20 mm Hg?

Encourage the client to hyperventilate Normal ICP is 15 mm Hg or less. Hyperventilation causes vasoconstriction, which reduces CSF and blood volume, two important factors for reducing a sustained ICP of 20 mm Hg. A cooling blanket is used to control the elevation of temperature because a fever increases the metabolic rate, which in turn increases ICP. High doses of barbiturates may be used to reduce the increased cellular metabolic demands. Fluid volume and inotropic drugs are used to maintain cerebral perfusion by supporting the cardiac output and keeping the cerebral perfusion pressure greater than 80 mm Hg.

A client arrives at the ER after slipping on a patch of ice and hitting her head. A CT scan of the head shows a collection of blood between the skull and dura mater. Which type of head injury does this finding suggest?

Epidural hematoma An epidural hematoma occurs when blood collects between the skull and the dura mater. In a subdural hematoma, venous blood collects between the dura mater and the arachnoid mater. In a subarachnoid hemorrhage, blood collects between the pia mater and arachnoid membrane. A contusion is a bruise on the brain's surface.

The nurse is caring for the client in the ER following a head injury. The client momentarily lost consciousness at the time of the injury and then regained it. The client now has lost consciousness again. The nurse takes quick action, knowing this is compatible with:

Epidural hematoma The changes in neurological signs from an epidural hematoma begin with a loss of consciousness as arterial blood collects in the epidural space and exerts pressure. The client regains consciousness as the cerebral spinal fluid is reabsorbed rapidly to compensate for the rising intracranial pressure. As the compensatory mechanisms fail, even small amounts of additional blood can cause the intracranial pressure to rise rapidly, and the client's neurological status deteriorates quickly.

The nurse is caring for a client with Guillain-Barré syndrome (GBS) who is receiving intravenous immunoglobulin (IVIG). Which assessment finding warrants immediate evaluation?

Headache with stiff neck Immediate evaluation is needed when a client with GBS receiving IVIG complains of a headache with stiff neck. This may be a sign of aseptic meningitis, a possible serious complication of IVIG therapy.Chills, generalized malaise, and a low-grade fever are minor adverse effects of IVIG therapy and do not indicate that the therapy must be stopped.

A client is admitted to the ER for head trauma is diagnosed with an epidural hematoma. The underlying cause of epidural hematoma is usually related to which of the following conditions?

Laceration of the middle meningeal artery Epidural hematoma or extradural hematoma is usually caused by laceration of the middle meningeal artery. An embolic stroke is a thromboembolism from a carotid artery that ruptures. Venous bleeding from the arachnoid space is usually observed with subdural hematoma.

A client with a spinal cord injury is prone to experiencing autonomic dysreflexia. The nurse would avoid which of the following measures to minimize the risk of recurrence?

Limiting bladder catheterization to once every 12 hours The most frequent cause of autonomic dysreflexia is a distended bladder. Straight catherization should be done every 4 to 6 hours, and Foley catheters should be checked frequently to prevent kinks in the tubing. Constipation and fecal impaction are other causes, so maintaining bowel regularity is important. Other causes include stimulation of the skin from tactile, thermal, or painful stimuli. The nurse administers care to minimize risk in these areas.

A client is admitted with a spinal cord injury at the level of T12. He has limited movement of his upper extremities. Which of the following medications would be used to control edema of the spinal cord?

Methylprednisolone (Solu-Medrol) High doses of Solu-Medrol are used within 24 hours of spinal injury to reduce cord swelling and limit neurological deficit. The other drugs aren't indicated in this circumstance.

A client with a T1 spinal cord injury arrives at the emergency department with a BP of 82/40, pulse 34, dry skin, and flaccid paralysis of the lower extremities. Which of the following conditions would most likely be suspected?

Neurogenic shock Loss of sympathetic control and unopposed vagal stimulation below the level of injury typically cause hypotension, bradycardia, pallor, flaccid paralysis, and warm, dry skin in the client in neurogenic shock. Hypervolemia is indicated by rapid and bounding pulse and edema. Autonomic dysreflexia occurs after neurogenic shock abates. Signs of sepsis would include elevated temperature, increased heart rate, and increased respiratory rate.

While in the ER, a client with C8 tetraplegia develops a blood pressure of 80/40, pulse 48, and RR of 18. The nurse suspects which of the following conditions?

Neurogenic shock Symptoms of neurogenic shock include hypotension, bradycardia, and warm, dry skin due to the loss of adrenergic stimulation below the level of the lesion. Hypertension, bradycardia, flushing, and sweating of the skin are seen with autonomic dysreflexia. Hemorrhagic shock presents with anxiety, tachycardia, and hypotension; this wouldn't be suspected without an injury. Pulmonary embolism presents with chest pain, hypotension, hypoxemia, tachycardia, and hemoptysis; this may be a later complication of spinal cord injury due to immobility.

During an episode of autonomic dysreflexia in which the client becomes hypertensive, the nurse should perform which of the following interventions?

Put the client in the high-Fowler's position Putting the client in the high-Fowler's position will decrease cerebral blood flow, decreasing hypertension. Elevating the client's legs, putting the client flat in bed, or putting the bed in the Trendelenburg's position places the client in positions that improve cerebral blood flow, worsening hypertension.

A 22-year-old client with quadriplegia is apprehensive and flushed, with a blood pressure of 210/100 and a heart rate of 50 bpm. Which of the following nursing interventions should be done first?

Raise the head of the bed immediately to 90 degrees Anxiety, flushing above the level of the lesion, piloerection, hypertension, and bradycardia are symptoms of autonomic dysreflexia, typically caused by such noxious stimuli such as a full bladder, fecal impaction, or decubitus ulcer. Putting the client flat will cause the blood pressure to increase even more. The indwelling urinary catheter should be assessed immediately after the HOB is raised. Nitroglycerin is given to reduce chest pain and reduce preload; it isn't used for hypertension or dysreflexia.

A client is admitted with an exacerbation of Guillain-Barré syndrome (GBS), presenting with dyspnea. Which intervention does the nurse perform first?

Raises the head of the bed to 45 degrees The nurse's first action for a client with an exacerbation of GBS who now has dyspnea is to raise the head of the bed to 45 degrees. The head of the client's bed must be elevated to allow for increased lung expansion. This action helps improve the client's ability to breathe.Calling the RRT for intubation may be necessary if dyspnea is severe or oxygen saturation does not respond to oxygen therapy. Close monitoring of respiratory status is indicated because of the acute stages of GBS. Instructing the client on how to cough effectively is not the priority in this case. The client would be suctioned as needed but cautiously to avoid vagal stimulation.

A client with a subarachnoid hemorrhage is prescribed a 1,000-mg loading dose of Dilantin IV. Which consideration is most important when administering this dose?

Rapid dilantin administration can cause cardiac arrhythmias. Dilantin IV shouldn't be given at a rate exceeding 50 mg/minute. Rapid administration can depress the myocardium, causing arrhythmias. Therapeutic drug levels range from 10 to 20 mg/ml. Dilantin shouldn't be mixed in solution for administration. However, because it's compatible with normal saline solution, it can be injected through an IV line containing normal saline. When given through an IV catheter hand, dilantin may cause purple glove syndrome.

Goals of Treatment of SCI?There are many. List 2-3

Stabilize area of injury Preserve neurological function and prevent disability Prevent secondary injury Initiate prophylaxis Pain management Prevent complications Address psychological and emotional needs Progress toward rehabilitation

Which are risk factors for stroke? (Select all that apply.) a. Use of oral contraceptives b. Female gender c. Previous stroke or transient ischemic attack d. High blood pressure e. smoking

a,c,d,e Common modifiable risk factors for developing a stroke include smoking and the use of oral contraceptives, specifically in women over the age of 35 and in women over the age of 30 who smoke. Other risk factors include high blood pressure and history of a previous TIA. Gender is not a known risk factor for stroke; however, the female client is at risk for delayed recognition of early stroke


Set pelajaran terkait

Chapter 1: Introduction to Leadership

View Set

Kinesiology - Hip, Knee, Ankle, Back (Chapters 8-11)

View Set

L21 Ready-to-use Therapeutic Foods (RUTF's)

View Set